Sei sulla pagina 1di 134

Universidad La Salle.

Facultad Mexicana de Medicina.


Curso de Extensin Universitaria para la Preparacin del Examen Nacional para
Aspirantes a Residencias Mdicas.
Examen Final 2011.
Modalidad a Distancia.
31 de Agosto del 2011.
1.-Femenino de 10 das de vida, es llevada a consulta externa por sus padres Antecedente:
obtenida por parto eutcico a las 41 semanas de gestacin. El peso inicial de la paciente se
encuentra por debajo de la percentila 2 y la circunferencia ceflica por debajo de la
percentila 4. Exploracin Fsica: cara redonda, baja implantacin de orejas y puente nasal
amplio, Trax se escucha llanto parecido a un maullido. Anormalidad cromosmica ms
probable en esta paciente?

a)

Delecin de 5p.

b)

Delecin de 3p.

c)

Trisoma 13.

d)

Trisoma 18.

Sndrome Cri du chat: Se produce una delecin terminal o intersticial del brazo corto del
cromosoma 5. Lo padece alrededor de 1% de los pacientes con retraso mental. Clnicamente
se distingue por microcefalia, hipertelorismo, pliegues epicnticos, orejas de implantacin
baja y micrognatia.
Thompson &Thompson, Gentica en Medicina, 7 Edicin, Editorial Elsevier, Pg. 93-95.

2.-Femenino de 23 aos, acude a su consultorio refiriendo

secrecin vaginal ftida.

Exploracin Fsica: Especuloscopa se observa secrecin gris que rodea la cpula vaginal. En
la preparacin en fresco se observan cocobacilos que rodean a las clulas epiteliales.
La medida teraputica ms apropiada en esta paciente es?

a)

Clindamicina

b)

Clotrimazol

c)

Metronidazol

d)

Tratamiento para ella y su pareja.


Metronidazol: Tratamiento especfico en vaginosis, la evidencia apoya en primer lugar el
uso de:
metronidazol por va oral, en dosis de 500mg cada 12 horas durante siete das, con una tasa
de curacin de 80 a 90 %. Para el tratamiento local no se ha demostrado una diferencia
significativa en la efectividad para curarla vaginosis, al comparar clindamicina en crema a 2
% y metronidazol vaginal, ni entre stos y el tratamiento con metronidazol por va bucal (Ia
y III).4,8 En vaginosis tampoco se ha demostrado la utilidad de tratar a la pareja sexual
(Ia).
Trejo

PJA,

Hernndez

LB,

Carrasco

RJR,

Ducoing

DDLR

Gua clnica para el diagnstico, tratamiento y prevencin de cervicovaginitis por bacterias,


Trichomonas y Candida Rev Med IMSS 2003; 41 (Supl 1):71-76.

3.- Femenino acude a su consultorio con diagnstico de anemia microctica e hipocrmica,


Resultados de laboratorio reportan: hierro srico y ferritina bajos, aumento de la
capacidad de fijacin del hierro y de la protoporfirina eritrocitaria, con hemoglobina A2
disminuida. La entidad responsable de los resultados de laboratorio de esta paciente es:

a)

Anemia por deficiencia de hierro.

b)

Anemia de los trastornos crnicos.

c)

Anemia sideroblstica.

d)

Talasemia beta.

Las causas de anemia ferropnica se pueden clasificar de acuerdo a su mecanismo de


produccin:
a) Por aumento de los requerimientos:
a1. Crecimiento acelerado:
- Primer ao de vida.
- Adolescencia.
- Embarazo.
- Prematurez.
a2. Aumento de las prdidas:


- Menstruacin.
- Hemorragia visible (por ejemplo: epistaxis a repeticin) u oculta (digestiva, renal, etc.).

b) Por aporte insuficiente:


- Dieta insuficiente.
- Reservas insuficientes al nacimiento:
Prematurez, gemelar, hemorragia intrauterina
(fetomaterna, intergemelar), etc.
- Alteraciones de la absorcin: sndromes de malabsorcin, resecciones de tubo digestivo,
etc.

Diagnstico y dx. Diferencial:

Anemia ferropnica. Normas de diagnstico y tratamiento / 163


Arch.argent.pediatr 2001; 99(2) / 162

4.- Femenino de 29 aos. Acude al servicio de urgencias por presentar salida de lquido
vaginal. Antecedentes: G2, P1, cursa embarazo de 36 semanas de gestacin exploracin
fsica: Crvix con 10% de borramiento, 1 cm de dilatacin y Tarnier positivo.
La complicacin ms frecuente en esta paciente es?
a)

Corioamnioitis.

b)

Parto pre trmino.

c)

Sepsis neonatal.

d)

Endometritis.

Corioamnioitis: El diagnstico de la infeccin intraamnitica (IIA) es bsicamente clnico.


La corioamnionitis se debe descartar en toda gestante que presente fiebre sin foco
aparente, sobre todo si se sospecha o se ha confirmado una rotura de membranas. Los
criterios ms empleados para el diagnstico son: fiebre materna y, al menos, 2 de los
siguientes signos: taquicardia materna, taquicardia fetal, irritabilidad uterina, leucocitosis
materna o lquido amnitico purulento o maloliente.

Progresos de obstetricia y ginecologa: revista oficial de la Sociedad espaola de


ginecologa y obstetricia, ISSN 0304-5013, Vol. 48, N. 6, 2005 , pags. 316-317.


5.- Debemos sospechar un retinoblastoma en un nio que presenta los siguientes sntomas:

a) Dolor, fotofobia y lagrimeo.


b) Lagrimeo, fotofobia y aumento del dimetro corneal.
c) Fotofobia y quemosis conjuntival.
d) Estrabismo y leucoria.

RETINOBLASTOMA
TUMOR OCULAR MS FRECUENTE EN INFANCIA.
1/20.000 RN
80% en < 3 aos
Uni o bilaterales.
Herencia
1. AD alta penetrancia (90-95%)
2. Espordicos

Diagnstico
Leucocoria
Estrabismo
Mala AV
Ojo rojo y doloroso
Celulitis orbitaria
Examen de rutina

1.-Annals d.Oftalmologia 2001;9(2):74-92


N. Martn, MD. Coll, J. Garca, J. Snchez de Toledo, E. Trivio, M. Guitart, JJ. Gil
2.-Unidad Oftalmologa Peditrica. Hospital Materno infantil Vall d.Hebron.
3.-Departamento de biologa-celular, fisiologa e inmunologa de la Universidad Autnoma
de Barcelona.
4.-Servicio oftalmologa Hospital General Vall d.Hebron.

6.- Masculino de 5 aos es llevado al servicio de urgencias por dolor sbito hiperemia, y
aumento de volumen en regin interna de rbita derecha. Exploracin Fsica: Se palpa masa
bien delimitada dolorosa en el borde interno de la rbita.
La medida teraputica inicial en este paciente es:

a)

Slo drenaje.

b)

Dicloxacilina y drenaje.

c)

Tobramicina tpico y drenaje.

d)

Extirpacin de saco lagrimal.

El tratamiento debe iniciarse antes de que se identifique el microorganismo causal. Tan


pronto se obtienen cultivos

de nariz, conjuntivas y sangre deben administrarse


antibiticos. La teraputica antibitica inicial debe cubrir estafilococos, H influenzae y
anaerobios. La mayor parte de los casos responde a los antibiticos. Aquellos en los cuales
esto no es as, pueden requerir drenaje.
Riordan P,Witcher j, Oftalmologa general de Vaughan y Asbury, 17 Ediciin, Pg. 262.

7.- Masculino de 65 aos el cual presenta adenopatas mltiples. El estudio morfolgico de


una de ellas muestra arquitectura borrada por la presencia de un componente celular
polimorfo, con abundantes clulas mononucleadas y binucleadas con nuclolo evidente que
son CD15+ y CD30+, entre ellas se disponen numeroso linfocitos CD3+, as como numerosos
eosinfilos histiocitos y clulas plasmticas. Cul es el diagnstico ms probable?

a)
b)
c)
d)

Linfoma B de clulas grandes anaplsico CD30+.


Linfoma T perifrico.
Enfermedad de Hodgkin tipo celularidad mixta.
Enfermedad de Hodgkin tipo predominio linfoctico.

Supone el 20-30% de todos los linfomas. Aparece con una discreta mayor incidencia en
varones que en mujeres, salvo en la variedad escleronodular que es ms frecuente en estas.
Desde el punto de vista clnico, la mayora de los casos debutan por afectacin ganglionar y
en el 75% de estos, el ndulo se localiza en regin cervical. Desde el punto de vista
epidemiolgico, esta enfermedad aparece extraordinariamente relacionada con el virus
Epstein-Barr.

En

este

sentido,

pacientes

con

antecedentes

de

haber

padecido

mononucleosis infecciosa, tienen de 2 a 4 veces con ms frecuencia enfermedad de Hodgkin


que la poblacin que no ha tenido contacto con el virus. Asimismo en pacientes con
enfermedad de Hodgkin, se pueden identificar ttulos altos de anticuerpos frente al virus
Reed Sternberg. Asimismo se han identificado fragmentos del genoma viral en el interior
de las clulas de Epstein-Barr. Si bien el papel exacto del virus Epstein-Barr en las gnesis
del Hodgkin es desconocida, todos estos datos apuntan una ntima relacin entre este virus
y la enfermedad de Hodgkin.
Desde el punto de vista histolgico, podemos definir la enfermedad de Hodgkin, como una
tumoracin compuesta por clulas de Reed-Sternberg y sus variantes (clulas de Hodgkin),
sobre un fondo de linfocitos pequeos, eosinfilos, neutrfilos, clulas plasmticas,
histiocitos y fibroblastos. En el 10% de los casos pueden encontrarse granulomas
epitelioides. Como ya se ha comentado, existen diferentes clasificaciones para esta
enfermedad, sin embargo de todas ellas la ms aceptada en la actualidad es la de Rye, que
clasifica esta enfermedad en 4 grupos: predominio linfoctico, esclerosis nodular,
celularidad mixta y "depleccin" linfocitaria.

Celularidad mixta:
Supone aproximadamente un tercio de los casos de enfermedad de Hodgkin y su patrn
histolgico es una mezcla de linfocitos, clulas plasmticas, eosinfilos, clulas de
Sternberg y clulas de Hodgkin.
CITOPATOLOGA DE LOS GANGLIOS LINFTICOS
Dr. Jos Mara Viguer,
Dra. Blanca Vicandi,
Dra. Pilar Lpez-Ferrer y Dr. Jos Antonio Jimenez-Heffernan.

8. - A 23-year-old nulligravid female has not menstruated in the past 4 months. Previously,
her menstrual cycles were regular. She is otherwise well and denies recent onset of
stress, change in exercise routine, headaches, visual field alterations, or galactorrhea. She
has a body mass index of 24, blood pressure of 120/78 mm Hg, and does not appear
hirsute. No adnexal masses can be palpated. Laboratory investigations reveal a negative
beta human chorionic gonadotropin (-hCG), normal thyroid-stimulating hormone (TSH),
and prolactin levels.
What is the next best step in the management of this patient?
a)

Measurement of luteinizing hormone.

b)

CT scan of the sella turca.

c)

Prescribe oral estrogen for 21 days followed by 7 days of medroxyprgesterona and

reevaluate.
d)

Preescribe 7 days of medroxyprgesterona and reevaluate.

La Prueba de respuesta a progestgenos se basa en la observacin de que el tratamiento


con progestgeno (acetato de medroxiprogesterona 10 mg por 5 a 6 das) solo induce la
menstruacin en las mujeres con concentraciones normales de estrgenos circulantes. Una
prueba positiva (hemorragia despus de concluir el tratamiento con progestgenos) seala
cifras normales de produccin de estrgenos y una prueba negativa (sin hemorragia por
privacin), hipogonadismo franco.
Danforth, Tratado de Obstetricia y Ginecologa, 9 Edicin, Ed. Mc Graw Hill
Interamericana, Pg. 668.

9. - Se trata de masculino de 55 aos alcohlico, que ingresa a la sala de urgencias por


sospecha de sndrome de Boerhaave o rotura espontnea del esfago. La triada clnica
clsica de sta patologa consiste en?

a) Nuseas, vmitos y fiebre.


b) Vmitos, dolor torcico y enfisema subcutneo.
c) Fiebre, dolor abdominal y hematemesis.
d) Dolor torcico, dolor abdominal y vmito.

Definicin: Ruptura espontnea del esfago que ocurre sobre todo como consecuencia de un
violento vmito.

Este sndrome fue descrito por primera vez en 1724 por Herman Boerhaave. Se
diferencia del sndrome de Mallory-Weiss en que en este caso se tratada de una
perforacin transmural mientas que el sndrome de Mallory-Weiss es tan solo una
laceracin. Dado que frecuentemente est asociado al vmito, no puede decirse que en
realidad se trate de una ruptura espontnea. Hay que distinguirlo de la perforacin
yatrognica que supone el 85-90% de los casos de rupturas esofgicas. Casi siempre en el
lado izquierdo del tercio inferior esofgico a 2 o 3 cm de la unin gastroesofgica. A igual
que el sndrome de Mallory-Weiss, se ha relacionado con los aumentos bruscos de la presin
intrabdominal y con la hernia de hiato. Es ms frecuentemente observado en pacientes
alcohlicos o que abusan de la comida.

Se trata de un sndrome relativamente raro pero con un alto ndice de mortalidad (35%).
De hecho, es considerada como la ms letal de todas las perforaciones del tracto digestivo.

Los sntomas son vmitos, dolor torcico y enfisema subcutneo cervical.


REFERENCIAS:
1.

C. Yeo. Shackelford' surgery of the Alimentary Tract, 2007. Saunders, 2700


pginas.

2. Khan, Aamir Z.; Forshaw, Mathew J.; Davies, Andrew R.; Youngstein, Taryn; Mason,
Robert C.; Botha, Abraham J. Transabdominal Approach for Management of
Boerhaave's Syndrome . American Surgeon , May2007, Vol. 73 Issue 5, p511-513.
3. Hill, Andrew G.; Tiu, Albert T.; Martin, Iain G. Boerhaave's syndrome : 10 years
experience and review of the literature: H. ANZ Journal of Surgery , Dec2003,
Vol. 73 Issue 12, p1008-1010.
4. Ochiai, T.; Hiranuma, S.; Takiguchi, N.; Ito, K.; Maruyama, M.; Nagahama, T.;
Kawano, T.; Nagai, K.; Nishikage, T.; Noguchi, N.; Takamatsu, S.; Kawamura, T.;
Teramoto, K.; Iwai, T.; Arii, S. Treatment strategy for Boerhaave's syndrome..
Diseases of the Esophagus , May2004, Vol. 17 Issue 1, p98-103.

10.- Paciente masculino de 52 aos con diagnstico de esquizofrenia desde hace 25 aos,
tratado con diversos neurolpticos en dosis elevadas. Hace dos aos inici con movimientos
repetitivos de labios y lengua, y movimientos coreoatetsicos de piernas y tronco que no
han mejorado a pesar de mltiples frmacos. Cul de estos factores es predisponente de
discinesia tarda?

a) Administracin prolongada de neurolpticos y anticolinrgicos.


b) El diagnstico de Esquizofrenia.
c) Una distimia agregada.
d) Una probable neoplasia cerebral.

Kaplan H I, Sadock B J. Sinopsis de Psiquiatra. Panamericana. 8 edicin. Madrid 2000. p.


711. Los factores predisponentes son la
pacientes mayores de 50 aos, y

edad avanzada, existe mayor probabilidad en

la administracin prolongada de neurolpticos y

anticolinrgicos, que ocurre varios meses o aos despus de iniciado el tratamiento con
agentes del bloqueo de la dopamina. Se cree que es por aumento de las concentraciones
sanguneas circulantes de dopamina, aumento de liberacin de dopamina desde las neuronas
dopaminrgicas, aumento de la capacidad de reaccin a la dopamina a nivel de sitios de
receptores postsinpticos y sntesis de nuevos receptores de dopamina de bajo umbral.

11.- Masculino de 33 aos, acude al servicio de urgencias por referir dolor en pabelln
auricular izquierdo, dificultad para mover la hemicara del mismo lado. Antecedentes: sin
importancia. Exploracin fsica: TA 120/80mmHg, FC 72 x, FR 16 x, Temp. 36 , se
observan vesculas en concha auricular y paresia de la hemicara izquierda.
El dato clnico que apoya el diagnstico en este paciente es:

a)

Parlisis de Bell.

b)

Parlisis facial perifrica.

c)

Parlisis facial central.

d)

Parlisis facial vascular.

Sndrome de Ramsay-Hunt: Representa 6.8% de las causas de parlisis faciales


intratemporales. La enfermedad se presenta en adultos entre 20 y 30 aos y 50 a 70 aos
de edad. Se caracteriza por la presencia de vesculas herpticas en pabelln auricular,
conducto auditivo externo y, con menor frecuencia paladar blando y cara. Por lo general se
inicia con otalgia intensa seguida en pocos das de la erupcin vesicular y parlisis facial
perifrica; adems puede haber hipoacusia, acfenos y vrtigo.

Escajadillo J, Odos, narz, garganta y ciruga de cabeza y cuello, Ed. Manual Moderno, 2
Edicin, Pg. 152.

12.- Femenino de 19 meses, es llevada por su madre al servicio de urgencias de Pediatra


por presentar exantema generalizado. Antecedentes: Irritabilidad, anorexia, y haber
recibido tratamiento a base de acetaminofen 48 hrs. previas a la consulta. Exploracin
fsica: Temp. 37.0, TA 100/65mmHg, FC 101 x, FR 26 x, exantema generalizado. En este
paciente el diagnstico ms probable es:

a)

Eritema infeccioso.

b)

Rosola.

c)

Rubola.

d)

Varicela.

Rosola: Es una enfermedad febril exantemtica leve, que ocurre casi en exclusiva durante
la lactancia.

El perodo prodrmico de la rosola suele ser asintomtico, pero puede

presentarse con signos leves del tracto respiratorio superior, entre ellos rinorrea mnima,
inflamacin farngea ligera y enrojecimiento conjuntival leve, adenoparias cervicales.
La enfermedad clnica suele ser precedida por temperatura alta, la fiebre persiste durante
3 a 5 das, y despus es tpica la resolucin brusca. El exantema aparece en 12-24 hrs.
siguientes a la resolucin de la fiebre, comienza en forma de lesiones rosadas, un poco
elevadas, pequeas, y delimitadas en tronco, y se suele extender al cuello, cara, y regiones
proximales de las extremidades, no suele ser pruriginoso.

Nelson, Tratado de Pediatra, 17 Edicin, Ed. Elsevier, Pg. 1069-1071.

13.- Femenino de 2 aos, es llevada al servicio de urgencias por presentar dolor abdominal
intermitente. Antecedentes: evacuaciones sanguinolentas en nmero de 6 en 24 horas de
48 hrs. de evolucin. Exploracin fsica: T/A 110/65mmHg, FC 90 x, FR 28 x, Temp.
36.5, irritabilidad, llanto fcil, se palpa masa dolorosa a nivel de colon ascendente.
En este paciente el diagnstico ms probable es:
a)

Divertculo de Meckel.

b)

Absceso apendicular.

c)

Invaginacin intestinal.

d)

Apendicitis aguda.

Invaginacin intestinal: Ocurre cuando una porcin del tracto alimentario se pliega dentro
del segmento adyacente. Es la causa ms frecuente de obstruccin intestinal entre los 3
meses y los 6 aos de edad. El cociente varn: mujer es 4:1. Se produce de forma sbita
con dolor clico paroxstico intenso recurrente a intervalos frecuentes, el 60% de los
lactantes expulsan heces que contienen sangre roja y moco, las deposiciones en confitura
de grosella.

Nelson, Tratado de Pediatra, 17 Edicin, Ed. Elsevier, Pg.1242-1243.

14.- Un lactante a trmino sano de 2 das de vida presenta petequias generalizadas y tiene
un recuento plaquetario de 15 000/mm. Su madre no tiene historia de enfermedades
anteriores y muestra un recuento sanguneo completo normal. Cul de los siguientes es el
diagnstico ms probable?

a)
b)
c)
d)

Prpura trombocitopnica idioptica adquirida pasivamente.


Enfermedad de von Willebrand.
Trombocitopenia autoinmunitaria.
Trombocitopenia amegacarioctica.

La trombocitopenia isoinmune resulta de la adquisicin pasiva de anticuerpos de la madre


dirigidos contra un antgeno de membrana plaquetaria del feto del que carecen las
plaquetas de la madre. El diagnstico debe sospecharse en un lactante sano
trombocitopnico cuya madre presenta un recuento de plaquetas normal,

y carece de

historia previa de prpura trombocitopnica idioptica o esplenectoma. La coagulopata de


consumo,

suele complicar el curso de un paciente grave.

La trombocitopenia

amegacarioctica es un trastorno muy raro asociado a anomalas del esqueleto como


ausencia del antebrazo.

LECTURA RECOMENDADA:
Boletn mdico del Hospital Infantil de Mxico versin impresa ISSN 1665-1146.
Bol. Med. Hosp. Infant. Mex. v.61 n.2 Mxico abr. 2004.
Bol Md Hosp Infant Mx 2004; Vol. 61(2):119-127.
ARTCULO ORIGINAL

15.- Femenino de 22 aos de edad, acude al servicio de urgencias por referir malestar
general, fosfenos, nusea y vmito. Antecedentes: cursa con embarazo de 36.5 SDG, G2,
A1, C0. Exploracin fsica: TA 185/110 mmHg, FC 120 lpm, FR 35 x, T37.9, alerta,
inquieta, aprecia ictericia, cardiopulmonar sin compromiso, abdomen globoso a expensas de
tero grvido, se detecta PUVI, longitudinal, ceflico, dorso a la derecha, FCF 142 x, al
tacto genital crvix, central, formado, cerrado, extraccin de guantes sin evidencia de
prdidas genitales, extremidades inferiores edema +++. El estudio de laboratorio que
confirma el diagnstico en esta paciente es:

a)
b)
c)
d)

Protena C reactiva.
Protenas en orina.
Protenas en suero.
Protenas hepticas.

Referencia:
La preeclampsia se define por aumento de la presin arterial y la presencia de proteinuria
durante el embarazo.
1)
Leve: las pacientes suelen tener unas cuantas manifestaciones, y su presin arterial
diastlica es menor de 110 mmHg. En ocasiones hay edema. La cifra de plaquetas es mayor
de 100 000/l.
2)
Grave: los sntomas son ms notorios y persistentes. La presin arterial casi
siempre es con niveles mayores de 160/110 mmHg. Puede haber trombocitopenia que avance
hasta coagulacin intravascular diseminada.
Se requiere hospitalizacin para las mujeres con preeclampsia; debe obtenerse una
biometra hemtica completa con cifra de plaquetas y determinacin de electrolitos, que
adems incluya enzimas hepticas. Se obtiene una muestra de orina de 24 horas para
determinar la depuracin de creatinina y protenas totales al ingreso hospitalario.
Se debe controlar la hipertensin arterial, para evitar sufrimiento fetal, as como
empeoramiento de la paciente.

Bibliografa:

1. McPhee S, Papadakis M, et. al. Diagnstico Clnico y Tratamiento 2010. Lange, McGraw
Hill, 49 edicin, Mxico, 2010.
2. Sibai BM, Diagnosis, prevention, and management of eclampsia. Obstet Gynecol. 2005.
Feb; 105;: 402 410.

16.- Femenino de 54 aos con datos clnicos de trombosis venosa profunda Cul de los
siguientes procedimientos es la primera prueba que debe solicitar el mdico para
establecer el diagnstico?

a) Cateterismo cardaco.
b) Venografa con contraste.
c) Pletismografa de impedancia.
d) Estudios de ventilacin y perfusin pulmonares.

La pletismografa de las extremidades inferiores es una prueba incruenta til para


establecer el diagnstico de trombosis venosa profunda. La venografa con medio de
contraste proporciona el diagnstico definitivo casi en cualquier caso; sin embargo, esta
prueba cruenta puede, de hecho, causar tromboflebitis en una minora de los casos. El
cateterismo cardaco, el gammagrama pulmonar y la tomografa computadorizada (CT) no
son tiles para diagnosticar trombosis venosa profunda.
Allen R. M. MMS Medicina Interna. 5. Edicin. National Medical Series. Mc. Graw Hill.
2006. (captulo 1 VIIIA 4 a-b),

17.- Masculino de ao y medio de edad, quien ha dejado de caminar, permanece quieto,


cadera y rodillas semiflexionadas, pies girados hacia fuera, Rosario en la unin
condrocostal, con franca depresin esternal, encas inflamadas de color prpura azulado,
con algunas petequias, Rx de huesos largos: huesos con aspecto de cristal esmerilado,
adelgazamiento de la cortical, lnea metafisiaria blanca gruesa e irregular, con zona de
rarefaccin por debajo y centros epifisiarios rodeados de un anillo blanco, Cul es su
diagnstico presuncional?

a)
b)
c)
d)

Escorbuto
Leucemia
Raquitismo
Prpura.

MANIFESTACIONES CLNICAS
Raquitismo
Los nios con raquitismo, a diferencia de los que sufren tantas otras enfermedades
carenciales, por lo general son rollizos y aparentan estar bien alimentados, debido a que su
consumo de energa es casi siempre correcto. Su aspecto engaa a menudo a la madre quien
piensa que todo est bien. Sin embargo, el nio se puede sentir indispuesto, y un examen
ms cuidadoso revelar la poca tonicidad muscular, que causa un abdomen protuberante.
Otra caracterstica del raquitismo es una alteracin general del desarrollo normal. El nio
se demora en alcanzar las etapas de la primera infancia, como la denticin aprender a
sentarse y caminar. Otros sntomas generales incluyen molestias gastrointestinales y
excesivo sudor en la cabeza.

Los signos principales de la enfermedad, y en los que se basa el diagnstico de raquitismo


son las deformaciones seas. La primera y principal caracterstica, es la hinchazn de los
extremos en crecimiento (epfisis), de los huesos largos. Esta inflamacin primero se puede
encontrar en la mueca, donde se afecta el radio. Otro sitio clsico es la unin de las
costillas con el cartlago costal; aqu la inflamacin tiene la apariencia de un rosario que se
conoce como rosario raqutico. Tambin se pueden observar hinchazones en los pies, la
tibia, el peron y el fmur. En los bebs con raquitismo la fontanela anterior se cierra
tardamente y en los nios mayores no es rara una protuberancia del frontal.
Una vez que un nio con raquitismo empieza a pararse, a caminar y estar activo, desarrolla
nuevas deformidades debido a la caracterstica blanda y dbil de los huesos. La deformidad
ms comn son las piernas en arco; con menos frecuencia se ven las rodillas juntas. Ms
serias, sin embargo, son las deformidades de la columna vertebral. Los cambios en la pelvis,
aunque raramente son visibles, pueden ocasionar dificultades en el parto a las mujeres que
han sufrido raquitismo en la infancia.
El raquitismo se puede diagnosticar por la apariencia clnica y radiolgica de los huesos, y
por exmenes de laboratorio.
Signos radiolgicos: Raquitismo
Alteraciones generales:
Retraso del crecimiento y osteopenia.
Alteraciones en el cartlago de crecimiento:
Primer signo radiolgico: ensanchamiento del cartlago de crecimiento.
Disminucin de la densidad de la zona de calcificacin provisional (del lado metafisario).
Desorganizacin del hueso esponjoso.
Ensanchamiento, deshilachamiento y "acopamiento" de las metfisis.
Espoln seo que rodea la fisis no calcificada.
Centros epifisarios irregulares y mal definidos.
Cortical borrosa.

Rosario raqutico: deformidad "en copa" de los extremos costales anteriores que se
presentan agrandados a la palpacin.
Zonas de Looser: Infrecuentes en nios.
Deformidades producidas por la debilidad sea:
Arqueamiento de los huesos largos: Tibia "en sable curvo".
Pelvis trirradiada: Protrusin acetabular bilateral + intrusin de la columna.
Surco de Harrison: Desplazamiento hacia adentro de la parte inferior del trax por
ablandamiento costal.
Escoliosis.
Vrtebras de pescado.
Invaginacin basilar.
Crneo tabes: Aplanamiento occipital y acumulacin de osteoide en las regiones frontal y
parietal.

BIBLIOGRAFA:
"Diagnstico por Imagen. Tratado de Radiologa Clnica". Csar S. Pedrosa.
"Diagnosis of Bone and Joint Disorders" Resnick.
"Trait de Radiodiagnostic. Tome X. Os: Pathologie Genrale". Raymond Trial.
"Gamuts en Radiologa". Stephen Chapman, Richard Nakielny.

18.- Femenino de 39 aos, acude a consulta refiriendo mareo a la bipedestacin.


Antecedentes: DM2 de 10 aos de evolucin bajo tratamiento mdico. Exploracin fsica:
TA 105/60mmHg sentada y de pie 95/60mmHg, FC 76 x, FR 16 x. Usted solicita ES.
Reportando: Na 164 mEq/l, osmolaridad urinaria 210 mOsm/l, osmolaridad srica 330
mOsm/l, sodio urinario 45 mOsm /l.
El diagnstico de primera eleccin en este paciente es:
a)

Diabetes juvenil

b)

Diabetes inspida nefrognica.

c)

Diabetes inspida central.

d)

Diabetes perifrica.

Diabetes inspida nefrognica: Se caracteriza por la falta de respuesta tubular a la AVP, en


presencia de concentraciones normales de la misma.
Causas: alteraciones del receptor V2, uso de litio, hipercalcemia, hipopotasemia,
enfermedades renales que interfieren en la concentracin de orina.
Consecuencias: Poliuria hipoosmtica (>40ml/Kg./d) (<200 mOsm/Kg.), tendencia a la
deshidratacin hipertnica (285-290mOsm/Kg.), con sodio de 140-145mEq/l, exceso de sed
en personas con conciencia conservada, la ingesta de sodio es igual a la secrecin
Hernando L, Nefrologa Clnica, Ed. Panamericana, 3 Edicin, 2008, Pg. 42-43.


19.-

Femenino de 8

aos presenta astenia, adinamia, decaimiento y prdida de 3

kilogramos de su peso habitual. Desde hace 15 das ha padecido fiebre de 39C, disfagia y
epistaxis. Al explorarla se encuentra plida, con adenomegalia cervical y axilar; se palpa
hepatoesplenomegalia y se observan quimosis en las piernas
practicar para establecer el diagnostico de certeza es:

a)
b)
c)
d)

Determinacin de Igm.
Biopsia ganglionar.
Prueba de Elisa para mycobacterium tuberculosis.
Biopsia de medula sea.

El estudio que se debe


La EH se presenta habitualmente con adenopatas cervicales o supraclaviculares, indoloras,
ms consistentes que las inflamatorias y de lento crecimiento.
Al menos, dos tercios de casos asocian adenopatas mediastnicas.

Sntomas sistmicos
Sntomas sistmicos no especficos pueden ser fatiga, anorexia y ligera prdida de peso.
Sntomas especficos (fiebre inexplicada, prdida de peso de 10% en los 6 meses previos y
sudor nocturno abundante) tienen significado pronstico.
Algunos pacientes tienen prurito.

Exmenes de laboratorio
En el hemograma, se observa leucocitosis con neutrofilia, linfopenia, eosinofilia y
monocitosis. La anemia puede indicar la presencia de enfermedad avanzada.
Se ha observado anemia hemoltica asociada a EH, con test de Coombs positivo,
reticulocitosis e hiperplasia normoblstica normoblstica de mdula sea. La asociacin de
prpura trombopnica idioptica (PTI) tambin ha sido referida y la respuesta al
tratamiento de la PTI va a depender del estadio en que se encuentre la EH. La elevacin de
VSG, cupremia y ferritinemia reflejan la activacin del sistema mononuclear fagoctico, que
puede servir como test inespecfico para control de seguimiento de la EH. Las anomalas del
sistema inmune en el momento del diagnstico de EH pueden persistir durante y despus
del tratamiento.

1.*** Fernndez-Teijeiro A. Enfermedad de


Hodgkin. En: Madero Lpez L y Muoz
Villa A editores. Hematologa y Oncologa peditricas. Madrid: Ergon; 1997.p. 467-82.
El autor hace un recuerdo histrico de EH, sealandoaspectos epidemiolgicos, etiolgicos
e histolgicos, resaltando la presentacin clnica y el diagnstico diferencial, as como los
estudios de diagnstico. Describe con amplitud aspectos teraputicos de radioterapia,
quimioterapia y de efectos secundarios del tratamiento.
2.* Ferrs Tortajada J, Garca Castell J, Berbel
Tornero O, Clar Gimeno S. Factores de riesgo para los linfomas no hodgkinianos.


An Esp Pediatr 2001; 55: 230-8.
Exposicin de factores de riesgo para LNH, revisados de la literatura mdica, con el
objetivo, segn los autores, de divulgar entre los pediatras dichos factores de riesgo.
3.* Ferrs Tortajada J, Garca Castell J, Lpez
Andreu JA, Berbel Tornero O, Clar
Gimeno S. Factores de riesgo para los linfomas de Hodgkin. An Esp Pediatric
2001; 55: 239-43.

20.- Masculino de 25 aos, cursando 24 horas de postquirrgico por drenaje de hematoma


subdural. Antecedentes: Traumatismo craneoenceflico. Exploracin fsica: T/A 110/70
mmHg, FC 78 x, FR 17 x, Temp 36.5. Sbitamente presenta crisis convulsiva tnicoclnica generalizada, la cual usted mitiga con un bolo de diacepam, al mismo tiempo solicita
laboratorios reportando: Na 127 mEq/l, K 4.0 mEq/l, Cl 97 mEq/l, glucosa 70 mg/dl,
creatinina 1.0 mg/dl.
El diagnstico inicial en este paciente es:
a)

Sx de Diabetes inspida.

b)

Sx hipoglucmico

c)

Sx de hipernatremia

d)

Sx de secrecin inapropiada de hormona antidiurtica.

Sndrome de secrecin inapropiada de hormona antidiurtica: Es la causa ms comn de


hiponatremia en pacientes hospitalizados. En un paciente que se presenta con
hiposmolalidad srica, pero con orina no diluida al mximo (>50 mOms/kg H2O en jvenes).
Criterios: Hipoosmolaridad (<300mmol/Kg) + hiponatremia, Osmolaridad urinaria >
Osmolaridad plasmtica, Sodio urinario semejante al ingerido, Ausencia de otras causas que
alteren la dilucin de orina (funcin renal, suprarrenal, tiroidea normales), Mejora de la
natremia a la restriccin acuosa, hormona antidiurtica (ADH) plasmtica no suprimida.
Test de sobrecarga de agua (evala osmolaridad plasmtica y urinaria), Ausencia de edema,
ortostatismo, deshidratacin.


Hernando L, Nefrologa Clnica, Ed. Panamericana, 3Edicin, 2008, Pg.40-41.

21.- Femenino acude a su consultorio con diagnstico de anemia microctica e hipocrmica,


Resultados de laboratorio reportan: hierro srico y ferritina bajos, aumento de la
capacidad de fijacin del hierro y de la protoporfirina eritrocitaria, con hemoglobina A2
disminuida. La medida farmacolgica a seguir en la paciente anterior si su padecimiento
fuera secundario a prdidas menstruales crnicas es:

a)

1000 a 2000 mg/da de Fe elemental en preparados de sales ferrosas v.o.

b)

10 a 20 mg/da de Fe elemental en preparados de sales frricas v.o.

c)

Complejo hierro-sorbitol por va i.m.

d)

100-200 mg/da de Fe elemental en preparados de sales ferrosas v.o.

En adultos, se ha demostrado que es necesario un aporte mnimo de 100 mg. a 200 mg. de
hierro elemento por da, para obtener la correccin total de la anemia y restablecer los
depsitos. El contenido de hierro elemento es muy variable en las formas comerciales, va
de 20 a 115 g. Aproximadamente 30mg de hierro se absorben diariamente cuando se
administran 180 mg. de hierro elemental. Luego, la dosis estndar debera ser de 60mg de
hierro elemental. Los efectos colaterales, tales como nuseas, dolores abdominales,
molestias epigtricas, constipacin y/o diarrea, se presentan en un 15 a 20% de los
pacientes, efectos que estn relacionados con la dosis.
Hillman R. Anemia ferropnica y otras anemias hipoproliferativas. En: Fauci A,
Braunwald E, Isselbacher K, Wilson J, Martin J, Kasper D, Hauser S, Longo D.,
eds.(1998) Harrisson Principios de Medicina Interna. 14 edicin, McGraw Hill,
Madrid.
22.- Femenino de 71 aos, acude a servicio de urgencias por presentar astenia y disnea.
Antecedentes: Insuficiencia renal crnica. Exploracin fsica: se observa con alteracin en
el estado de conciencia, ciantica y con ingurgitacin yugular. Temp. 37.7 C, TA 85/65
mmHg, FC 105, FR 17, a la inspiracin se observa disminucin de la presin sistlica de 15
mm Hg.
Qu dato espera encontrar en la exploracin fsica?
a)

Hepatomegalia.

b)

Ruidos cardiacos distantes.

c)

Masa abdominal pulstil.

d)

Egofona.

TAPONAMIENTO CARDIACO
Es la compresin cardaca provocada por derrame de lquido o acumulacin de
sangre en el pericardio. Es el aumento de la presin intrapericrdica que origina aumento de
la presin venosa y colapso circulatorio. El aumento de presin intrapericrdica ocurre
cuando hay un incremento en forma rpida del lquido percrdico por arriba de 60 a 80
mml.
SIGNOS AGUDOS:
Triada de Beck: Presin arterial baja, Elevacin de la presin venosa central, Ruidos
cardiacos apagados.

La injurgitacin yugular puede estar ausente debido a hipovolemia.

Paciente estuporoso, agitado

Extremidades fras y hmedas

Taquipneay taquicardia

Braunwald E. Tratado de cardiologa. 4ta edicin. Madrid : Interamericana,

McGraw-Hill : vol 2 :1645-1652

23.- A 55-year-old female is brought to the emergency room with a sudden onset of
severe chest, back, and abdominal pain that began 20 minutes ago. The pain is described as
being very sharp with a tearing-like character. Her previous medical history is notable for
hypertension treated with hydrochlorothiazide. On examination, her blood pressure is
145/95 mm Hg in the right arm and 119/75 mm Hg in the left arm; pulse, 105
beats/minute; temperature, 37.3C.; and respirations, 17 breaths/minute. A chest X-ray
(CXR) demonstrates a widened mediastinum, and electrocardiography (ECG) reveals
nonspecific ST and T wave changes.


What is the most appropriate next step in the management of this patient?
a)

Emergent surgery

b)

Thoracic computed tomography

c)

Intravenous labetalol

d)

Aortography

DISECCIN DE AORTA:

Los predictores clnicos de mayor sensibilidad para el diagnstico son: dolor

torcico sbito e intenso, ausencia o diferencia de pulsos y ensanchamiento mediastinal.

El manejo mdico est dirigido a disminuir la presin arterial media y de manera

muy importante, a disminuir el aumento de la presin pulstil que golpea directamente la


aorta (dp/dt).

Las medidas a tomar para este objetivo incluyen reposo absoluto, control de las

cifras de presin arterial, administracin de oxgeno y monitorizacin completa no invasiva.

Los objetivos iniciales de reduccin de las cifras de tensin sistlica deben

alcanzar los 100-120 mm Hg o menos, manteniendo la perfusin a los rganos vitales.

Los medicamentos antihipertensivos de eleccin son los beta bloqueadores IV,

combinados con nitroprusiato de sodio. Una terapia alternativa a esta combinacin, es el


uso de labetalol que tiene efecto bloqueador alfa y beta.

Khan IA, Chandra N. Clinical, Diagnostic, and management perspectives of aortic

dissection. Chest 2002; 122: 311-328.

24.- Femenino de 56 aos, se presenta al servicio de urgencias por presentar dolor lumbar
de 4 das de evolucin. Tiene antecedente de lumbalgia crnica. Niega traumatismo o cadas
recientes. A la exploracin fsica se observa que la paciente presenta dificultad para la
marcha, imposibilidad para elevar los pies y prdida del tono anal.
Cul es el tratamiento ms adecuado para esta paciente?
a)

Terapia de rehabilitacin.

b)

AINE y reposo.

c)

Radioterapia.

d)

Ciruga.

SINDROME DE CAUDA EQUINA:

Ocurre cuando las races nerviosas en la base de la columna vertebral se

comprimen. Conocida como cola de caballo (cauda equina, en latn), este grupo de nervios
determina la sensacin y funcionamiento de la vejiga, intestinos, rganos sexuales y
piernas.

Es una emergencia mdica. Si la ciruga no se realiza inmediatamente para aliviar la

presin sobre los nervios, podra perderse la funcin abajo de la cintura.


La ciruga podra involucrar:

Laminetoma

Disquectoma

Cauda equina syndrome. American Association of Neurological, October 31, 2008.

25.- Masculino de 39 aos, acude a consulta por presentar desde hace tres das tos
productiva, dolor torcico, disnea y fiebre. Sin antecedentes patolgicos de importancia.
A la exploracin fsica matidez sobre el hemotrax derecho y estertores gruesos
en la misma rea. Signos vitales: TA 120/80 mmHg, FC 115, FR 28, Temp. 39.1 C.
Radiografa de trax con imagen compatible a consolidacin en lbulo superior derecho.


Cul es el siguiente paso en el manejo de este paciente?
a)

Azitromicina va oral.

b)

Cultivo de esputo.

c)

Penicilina G benzatnica IM.

d)

Hemocultivo.

NEUMONIA ADQUIRIDA EN LA COMUNIDAD:

A) Paciente en tratamiento ambulatorio:


a. Paciente sin comorbilidades, sin uso crnico de antibiticos en los tres meses previos.
I. Primera opcin: macrlido. Si existe evidencia de encontrarse en una zona con alto ndice
(> 25%) de Streptococcus pneumoniae resistente a los macrlidos, se podr utilizar una
fluoroquinolona respiratoria (moxifloxacina, gemifloxacina)
II. Segunda opcin: doxiciclina
b. Paciente con alguna de las siguientes comorbilidades: Enfermedad crnica cardiaca,
pulmonar, heptica y/o renal; inmunodepresin por enfermedad o por uso de medicamentos;
uso de antimicrobianos en los tres meses previos
I. Primera opcin: Fluoroquinolona respiratoria (moxifloxacina, gemifloxacina)
II. Primera opcin: Un lactmico (cefotaxima, ceftriaxona, o ampicilina/sulbactam) en
combinacin con un macrlido.

Armitage K, Woodhead M. New guidelines for the management of adult community-

acquired pneumonia. Curr Opin Infect Dis. 2007;20:170-176.


26.- Masculino de 65 aos, acude a consulta por presentar desde hace 3 meses fatiga,
prdida de peso, vmito con sangre, saciedad temprana y dolor en epigastrio. El dolor no se
relaciona con los hbitos alimenticios. Exploracin fsica sin datos patolgicos aparentes.
Biometra hemtica con datos de anemia hipocrmica microctica.
Cul es el primer estudio que usted solicita?
a)

Endoscopa.

b)

Serie esfagogastroduodenal.

c)

Colonoscopa.

d)

Tomografa computada.

CANCER GASTRICO:

Los sntomas ms frecuentes de cncer de estmago son: dolor epigstrico, e

indigestin, anorexia, prdida ponderal, vmitos o hematemesis, melena, disfagia, lesin


ocupante del abdomen, diarrea y estaeatorrea.

La serie esfago-gastro-duodenal con doble contraste es el mtodo idneo para

estudiar el estmago a nivel de atencin primaria de la salud. La exactitud global de la


SEGD es mayor al 80%.

Tierney L, Diagnstico clnico y tratamiento, 35 Edicin, Tumores malignos del

estomago, Pg. 585-586, Manual Moderno.

27.- Femenino de 25 aos, acude a urgencias por cuadro de 3 das de evolucin


caracterizado por dolor en cuadrante superior derecho, de inicio insidioso, intermitente,
opresivo, sin irradiacin y de intensidad 7/10. Ingiere paracetamol para cefalea ocasional y
anticonceptivos orales desde hace varios aos. El ultrasonido demuestra una imagen bien
delimitada, hiperecica de 2 cm. de dimetro en el lbulo heptico derecho.
Cul es el diagnstico ms probable en esta paciente?
a)

Carcinoma hepatocelular.

b)

Quiste heptico.


c)

Adenoma heptico.

d)

Metstasis heptica.

Los adenomas hepticos (AH) son proliferaciones benignas de hepatocitos, que suelen
presentarse en mujeres entre los 20 y los 40 aos de edad y con antecedentes de toma de
anticonceptivos orales. Infrecuentemente se presentan en varones, siendo la proporcin de
varones: mujeres de 1:11.
El sntoma ms frecuente es el dolor abdominal vago crnico, que si se localiza en el
cuadrante superior derecho suele ser por una hemorragia intratumoral.
Actualmente, las tcnicas de imagen con mejor rendimiento para su diagnstico son la TC
helicoidal multifsica y la resonancia magntica (RM).
Hugh TJ, Poston GJ. Benign liver tumors and masses. Surgery of the liver and biliary tract.
3 edicin. London, 2000; tomo 2 p.1397-1422

28.- Masculino de 56 aos, indigente, es trado por paramdicos quienes refieren tos
purulenta y mal oliente. A su ingreso al servicio de urgencias saturacin de oxgeno de 72%.
En radiografa de trax se observan opacidades en lbulos medio e inferior de pulmn
derecho.
Qu tratamiento est indicado en este paciente?
a)

Cefuroxime IV.

b)

Metronidazol IV.

c)

Ceftriaxona IV.

d)

Clindamicina IV.

La neumona por aspiracin se desarrolla despus de la inhalacin de material orofarngeo


colonizado. La aspiracin de secreciones colonizadas de la orofaringe es el mecanismo
primario por el cual la bacteria gana entrada a los pulmones.

Los signos de hipoxemia, como la taquipnea, taquicardia y cianosis, surgen en forma


inmediata y persisten durante varias horas. La auscultacin torcica indica la presencia de
jadeos, estertores y roncus, y es factible que el paciente expectore grandes volmenes de
esputo sanguinolento y espumoso. La broncoaspiracin grave da por resultado insuficiencia
respiratoria con una combinacin de acidosis metablica y respiratoria.
Los antibiticos de eleccin para los casos de neumona por aspiracin son la penicilina y la
clindamicina
Georges CB. Neumona por broncoaspiracin, empiema y absceso pulmonar. En: Tintinalli JE,
Krome RL, Ruiz E. Medicina de Urgencias. 4 edicin. Interamericana McGraw Hill. Mxico.

29.- Masculino de 16 aos de edad, acude a consulta por haber presentado herida
cortocontundente en brazo derecho mientras trabajaba en el campo. Desconoce su
esquema de vacunacin. La herida est altamente contaminada.
Cul es el manejo ms apropiado de este paciente?

a)

Vacuna.

b)

Vacuna e inmunoglobulina.

c)

Inmunoglobulina.

d)

Slo lavado con agua y jabn.

El ttanos es una enfermedad aguda, caracterizada por la presencia de espasmos


musculares intensos e intermitentes y rigidez generalizada, secundarios a la accin de una
potente neurotoxina, conocida como tetanospasmina, elaborada por Clostridium tetani.
La puerta de entrada est dada por efracciones de la piel, mucosas y otras veces
directamente del msculo. Se distinguen cuatro perodos clnicos. Perodo de incubacin: es
frecuentemente de 5 a 15 das. Perodo de invasin: el trismo, que suele aparecer en este
perodo, es lo que evoca con seguridad el diagnstico de ttanos. Perodo de estado: se
producen contracturas musculares que se generalizan, son descendentes y dolorosas. El
diagnstico de ttanos es clnico.
El tratamiento consiste en erradicar al Clostridium tetani de los tejidos afectos con
tratamiento quirrgico y antibitico, la neutralizacin de la toxina circulante con
inmunizacin activa y pasiva y el tratamiento sintomtico.
Profilaxis: se utilizan las vacunas DT (antidiftria, antitetnica), DPTa (antidiftrica,
antibordetella pertusis, antitetnica) y la TIG (inmunoglobulina humana antitetnica).
Gorodner JO. Enfermedades infecciosas. 2nd ed. Buenos Aires: Corpus, 2002: 181-191.

30.- A 55-year-old woman presents to the Emergency Department with 8 hours of severe
left lower quadrant abdominal pain, focal peritonitis, and guarding on physical examination.
She has nonspecific abdominal radiographs and a leukocytosis.
Which of the following is indicated at this point?
a)

IV antibiotics.

b)

CT scan.

c)

Colonoscopy.

d)

Surgery.

ABSCESO DIVERTICULAR:
La formacin de un absceso diverticular complicado depende de la capacidad de los
tejidos periclicos de controlar (localizar) la diseminacin del proceso inflamatorio.
En general, los abscesos intra-abdominales se forman por:
o Fuga anastomtica = 35%
o Enfermedad diverticular = 23%

Signos/Sntomas

fiebre+/- leucocitosis a pesar de antibiticos adecuados, tumoracin dolorosa


Tratamiento:

Absceso periclico pequeo - 90% responde a los antibiticos y manejo


conservador.

Drenaje percutneo de los abscesos (DPA) es el tratamiento de eleccin para las


colecciones simples, bien definidas.

100%

de

los

abscesos

uniloculares

simples

se

resolvieron

con

DPA

antibiticoterapia.
Schauer P, Ramos P, Ghiatas A, Sirinek K. Virulent diverticular disease in young obese
men.Am J Surg;164:443-8. Pubmed-Medline

31.- Femenino de 2 aos, es llevada a urgencias refiriendo la madre llanto enrgico al


orinar. Antecedentes: Manejo con antibitico por cuadro de IVU previo, 2 semanas despus
los estudios complementarios de imagen muestran reflujo vesicouretral grado II.
El tratamiento a corto plazo de esta paciente debe incluir:
a) Cultivo de orina.
b) Antibitico profilctico.
c) Ciruga.
d) Cistoureterografa miccional cada 6-8 semanas.

El tratamiento depender del tipo de reflujo y graduacin del mismo. Habitualmente el RVU
grado 1 y grado 2 remiten espontneamente con el crecimiento del nio.Debern recibir
profilaxis antibitica y realizar controles mensuales de urocultivo durante los primeros
tres meses, luego en forma bimestral hasta el sexto mes y luego cada tres meses hasta los
2 aos de producida la infeccin urinaria.El RVU grado 3 requiere iguales controles como
los mencionado previamente pero con la salvedad de que no remiten espontneamente.
Aproximadamente en ms del 50% de los nios ser necesaria la correccin quirrgica. En
el RVU 4 y 5 la conducta teraputica habitual es la correccin quirrgica.

Tanagho E, Mc Aninch J, Urologa general de Smith, Manual Moderno, 14 Edicin,


p.p. 210-211

32.- Femenino de 25 aos, se presenta a consulta externa refiriendo aumento excesivo de


peso desde hace 6 meses a la fecha, disminucin de lbido, sensacin de tristeza y
amenorrea de 12 semanas. Exploracin fsica: Obesidad truncal y estras en abdomen. TA
150/100mmHg, FC 85 x, FR 16 x, Temp 37.3C. Laboratoriales: PIE negativo.
El siguiente paso para confirmar el diagnstico de esta paciente es:

a)

Prueba de tolerancia a la glucosa.

b)

Cortisol plasmtico.

c)

Prueba nocturna de supresin a la dexametasona.

d)

Potasio srico.

Prueba de supresin con dexametasona:


Este procedimiento se utiliza para establecer la presencia de sndrome de Cushing.
La dexametasona, glucocorticoide potente de manera normal suprime la liberacin
hipofisiaria de ACTH con una cada en los corticosteroides del plasma y orina, y as evala
la retroalimentacin inhibidora del eje HHS. En el sndrome de Cushing este mecanismo es
anormal y la secrecin de esteroides es incapaz de suprimirse en forma normal. La prueba
de supresin noturna con 1 mg de dexametasona, es un estudio adecuado para la deteccin
de sndrome de Cushing.
Se administra 1mg de dexametasona VO, como dosis nica a las 11 pm. y se toma una
muestra de plasma en la maana siguiente para determinar el cortisol. Si el valor es > de 10
ug/dl, la causa probables es sndrome de Cushing.
Gardner D, Shoback D, Endocrinologa bsica y clnica de Greenspan, Manual Moderno, 7
Edicin, pags. 378-381.

33.- Femenino de 33 aos gestante, acude a ultrasonido de control. Reportando: Placenta


marginal, embarazo de 37 SDG. Antecedentes: G=V P= IV.
La complicacin esperada en esta paciente es:

a)

Hemorragia materna.

b)

Desprendimiento de placenta normoinserta.

c)

Parto pretrmino.

d)

Restriccin en el crecimiento intrauterino.

Placenta previa: La placenta en lugar de estar implantada en el cuerpo del tero por fuera
del orificio cervical interno, se localizada por encima del orificio interno o cerca de l. En la
placenta marginal el borde la placenta est en el margen del orificio. La multiparidad y la
edad avanzada parecen favorecer la placenta previa. El hecho ms caracterstico de la
placenta previa es la hemorragia no dolorosa, la cual no suele aparecer hasta casi del final
de segundo trimestre o ms tarde. Complicaciones maternas: Hemorragia y muerte.

Pritchard, Williams Obstetricia, 3Edicin, Salvat editores, pag. 395-399

34.- Masculino de 7 aos, es trado a urgencias por cuadro diarreico desde hace 24 horas.
La diarrea es lquida, profusa, sin moco ni sangre, se acompaa de vmito de contenido
gstrico. Refiere la madre que varios de los compaeros de la primaria han presentado el
mismo cuadro.
El agente causal responsable del cuadro clnico de este paciente es:

a) Virus Norwalk.
b) E. coli
c) C. jejuni
d) Bacillus cereus

El virus del Norwalk pertenece al gnero de virus ARN de la familia Calciviridae, la


transmisin ocurre va fecal-oral. Agente infeccioso ms comn de las gastroenteritis
benignas de la comunidad. El promedio de incubacin es de 24 horas. El cuadro clnico por lo
comn dura de 12 a 60 horas y se caracteriza por uno o ms de los sntomas siguientes:
nusea, vmito, clicos abdominales y diarrea. El vmito es ms prevalente en nios, en
tanto que una proporcin mayor de los adultos mostrar diarrea.
Harrison, Principios de Medicina Interna, 16 Edicin, Mc Graw Hill, p.p. 1266-1267

35.- A 74-year-old man has developed a progressively worsening dry cough over the last 5
days. This has been accompanied by abdominal pain, diarrhea, headache, malaise, chills, and
fevers to 40.5C. On examination, there are extensive rales on chest auscultation.
Abdominal examination is normal. A urinary antigen test is positive and confirms the
presence of a gram-negative bacillus infection.
What is the most likely form of transmission of this patient's condition?

a)

Aerosolization.

b)

Fecal-oral.

c)

Hematogenous.

d)

Person to person.

Legionella. Es un bacilo gramnegativo. La transmisin est asociada a contaminacin de agua


de los sistemas de ventilacin, refrigeracin y calefaccin. Es ms frecuente en ancianos y
en inmunocomprometidos. Los sntomas tienden a empeorar durante los primeros 4 a 6 das
y normalmente mejoran en otros 4 a 5 das y pueden abarcar dolor torcico, hemoptisis,
fiebre, sntomas gastrointestinales, tales como diarrea, nuseas, vmitos y dolor abdominal.
Kumate J, Onofre M, Manual de Ifectologa clnica, Decimosexta edicin, 2001, Mndez
editores, Pg. 824-835.

36.- Femenino de 72 aos,


antecedentes:

Acude a

consulta por disnea de pequeos esfuerzos

diabetes mellitus en tratamiento y control medico,

infarto de miocardio

hace un ao, tiene una fraccin de eyeccin ventricular izquierda de 0,30 y est en
tratamiento habitual con aspirina, furosemida, (20 mg/da) y captopril, (25 mg/da))
exploracin fsica: Ingurgitacin Yugular , cianosis distal en manos. TA: 140/70 mmHg, FC:
94 lpm, edema pretibial mpi, ECG con datos de F.A.
La medida teraputica inicial en esta paciente es:

a) Iniciar el captopril por lisinopril (20 mg/da).


b) Iniciar tratamiento con digoxina (0,25 mg/da).
c) Iniciar tratamiento con espironolactona (25 mg/da).
d) Iniciar tratamiento con bisoprolol (1,25 mg/da).

Glucsicos cardiacos (digital): ICC


El tratamiento con digoxina debe aadirse a diurticos e IECAs (tratamiento triple),
siempre que no exista contraindicacin, para reforzar su eficacia cuando la respuesta
teraputica en la IC sistlica es subptima y persisten los sntomas. La digoxina est
especialmente aconsejada en pacientes con IC y fibrilacin auricular con respuesta
ventricular rpida, y en la disfuncin sistlica en clase funcional III-IV de la NYHA en
ritmo sinusal, en tanto que en la disfuncin ventricular asintomtica en fibrilacin auricular
pueden ser alternativas igualmente vlidas algunos calcioantagonistas o los
betabloqueantes.
El principal efecto de la digoxina es inotrpico (slo manifiesto en presencia de IC), aunque
probablemente tambin sea vasodilatador y diurtico. Los ensayos clnicos ms
recientes19 no han podido demostrar un aumento de supervivencia en enfermos con IC
tratados con digital, aunque confirman que mejora la situacin hemodinmica y la capacidad
funcional, pudiendo resultar til para aliviar la sintomatologa, prevenir el deterioro clnico
y reducir ligeramente la morbilidad. En contrapartida, aunque no existe un aumento de las
cifras de mortalidad global, s que se ha constatado un incremento de la mortalidad de
causa arritmognica en enfermos tratados con digoxina.

37.- Masculino de 66 aos que cursa con postinfarto agudo de miocardio una medicacin
generalmente indicada, por disminuir la mortalidad, es:

a)

Betabloqueadores.

b)

Nitritos.

c)

Anticoagulacin oral.

d)

Antiarrtmicos.

Los BB son recomendados en todos los pacientes que han presentado un IAM,

siempre que no tengan contraindicaciones para su uso, y de modo permanente


(indefinidamente): clase de recomendacin I, nivel de evidencia A.

Se ha puesto en evidencia que los BB son infrautilizados en esta indicacin.

Ms de 35.000 pacientes han sido incluidos en estudios postinfarto con BB.

Se ha demostrado una reduccin de la mortalidad total, muerte sbita y reinfarto

del orden del 20-25%.


Clase de

Nivel de

recomendacin

evidencia

Para mejorar supervivencia

Para prevenir reinfarto

Prevencin primaria de la muerte sbita

IIb

Situacin clnica/indicacin
Todos los pacientes sin
contraindicaciones, indefinidamente

Prevencin/tratamiento de arritmias
ventriculares tardas

Los BB siguen estando indicados en el tratamiento de la HTA, aunque existen

crticas

su

empleo

en

el

primer

escaln.

Pese a ello, son sin duda de primera eleccin si el/la paciente presenta angina, cardiopata
isqumica en general, insuficiencia cardaca, taquiarritmias, glaucoma o embarazo.

Los BB son frmacos de primera eleccin en la insuficiencia cardaca con disfuncin

sistlica. Slo el carvedilol, el bisoprolol o el nebivolol son BB que puedan ser empleados a
las dosis disponibles en nuestro medio para dicho tratamiento.

Todos los pacientes sin contraindicaciones deben recibir BB si presentan

cardiopata isqumica y, muy especialmente, si existe angina o han presentado un IAM.

ESC Expert consensus document on b-adrenergic receptor blockers. The Task Force on
Beta-Blockers

of

the

European

Society

of

Cardiology.

Eur Heart J 2004; 25: 13411362.

38.-Masculino de 62 aos,

refiere disnea de moderados esfuerzos y se le ausculta un

primer tono fuerte, chasquido de apertura y soplo diastlico con refuerzo presistlico y en
el ECG presenta ondas P con signos de crecimiento de la aurcula izquierda. El diagnstico
ms probable es:

a)

Doble lesin mitral en ritmo sinusal.

b)

Insuficiencia artica en ritmo sinusal.

c)

Estenosis mitral en ritmo sinusal.

d)

Estenosis mitral en fibrilacin auricular, probablemente severa.

Estenosis mitral:

Los hallazgos se concentran en el distole:

- Despus del 2 ruido se ausculta un chasquido de apertura (momento en que se abre la


vlvula mitral)

- Luego viene un soplo en decrescendo ("rodada mitral"): corresponde a la fase de llenado


rpido.

- A continuacin, en pacientes en ritmo sinusal se puede escuchar un refuerzo presistlico,


que es un soplo en crescendo y que se escucha inmediatamente antes del primer ruido (se
debe a la contraccin de las aurculas).

- El primer ruido, tiende a ser de mayor intensidad (la explicacin sera que cuando se
comienzan a contraer los ventrculos al comienzo del sstole, los velos de la vlvula mitral se
encuentran en su mxima separacin ya que la aurcula estaba recin contrayndose, y esto,
unido a las caractersticas de los velos, hara que el primer ruido sea ms intenso; sera
equivalente a un "portazo").
Estos ruidos y soplos se auscultan mejor en el pex. La auscultacin puede resultar ms
ntida si se pone a la persona en decbito semilateral izquierdo y se le pide que bote el aire
y se mantenga un rato sin respirar. Convendra auscultar tanto con la membrana como con la
campana del estetoscopio.

Si la estenosis mitral ha generado hipertensin pulmonar secundaria, el cierre de la vlvula


pulmonar (P2) es ms intenso y el segundo ruido se ausculta desdoblado porque se retrasa
el cierre de la vlvula pulmonar. Tambin puede ocurrir que el ventrculo derecho se
hipertrofie y sea palpable.
Cuando el soplo de "estenosis mitral" se debe a la inflamacin de los velos por una
enfermedad reumtica activa recibe la denominacin de soplo de Carey-Coombs.
Manual de semiologa .2007
Universidad Catlica de Chile

39.- Se trata de masculino de 61 aos, fumador de 10 cigarrillos diarios, historia de tos y


expectoracin matutina habitual, consulta por disnea de mnimos esfuerzos y ortopnea de
dos almohadas. Exploracin fsica: TA 180/100 mmHg, presin venosa normal, auscultacin
pulmonar con crepitantes bibasales, auscultacin cardaca rtmica a 120 lpm. con soplo
sistlico eyectivo I/VI en foco artico y tercer ruido. ECG: ritmo sinusal y criterios de
hipertrofia ventricular izquierda. El diagnstico ms probable es:

a)

Cardiopata isqumica con disfuncin sistlica.

b)

Cardiopata hipertensiva en insuficiencia cardaca.

c)

Cor pulmonale crnico.

d)

Insuficiencia cardaca congestiva en paciente con EPOC.


La cardiopata hipertensiva constituye la complicacin principal de la hipertensin arterial,
pues es la primera causa de morbi-mortalidad del paciente hipertenso. En la hipertensin
arterial la composicin histolgica del ventrculo izquierdo se altera globalmente,
resultando lesiones que afectan a los propios cardiomiocitos, al intersticio miocrdico y a la
pared de las arterias intramiocrdicas. En el origen del desarrollo de esas lesiones
participan tanto la sobrecarga mecnica de la pared ventricular impuesta por la presin
arterial elevada, como factores humorales sistmicos y locales que actan directamente
sobre el parnquima y los vasos miocrdicos, por ejemplo, la angiotensina II. Las
consecuencias funcionales de las lesiones estructurales miocrdicas son diversas, aunque la
ms representativa es la que tiene que ver con el desarrollo de insuficiencia cardiaca
congestiva
La insuficiencia cardiaca (IC) es un sndrome clnico complejo en el que los pacientes
presentan sntomas tpicos de IC (disnea en reposo o con esfuerzo), signos tpicos de IC
(taquicardia, taquipnea, estertores pulmonares, derrame pleural, elevacin de la presin
yugular venosa, edema perifrico, hepatomegalia) y evidencia objetiva de una anomala
estructural o funcional del corazn en reposo (cardiomegalia, tercer tono, soplos cardacos,
anomalas electrocardiogrficas, concentraciones elevadas de pptidos natriurticos) (ESC,
2008).
Cardiopata hipertensiva
MARIO BENDERSKY*, DANIEL PISKORZ#, DANIEL BOCCARDO
* Profesor de Farmacologa, Facultad de Ciencias Mdicas, Universidad Nacional de
Crdoba. Unidad de Hipertensin Arterial,
Instituto Modelo de Cardiologa de Crdoba.
# Instituto de Cardiologa del Sanatorio Britnico de Rosario.
Escuela de Cardiologa, Universidad Catlica de Crdoba. Departamento de Tcnicas No
Invasivas y Arritmias, Instituto Modelo
de Cardiologa de Crdoba.
Direccin postal: Instituto Modelo de Cardiologa. Av. Sagrada Familia 359. 5003 Crdoba.
Argentina.
Sanatorio Britnico de Rosario. Paraguay 40. 2000 Rosario. Pcia. de Santa Fe. Argentina.


40.- Masculino de 22 aos de edad, que acude al servicio de cardiologa, sin antecedentes
patolgicos y sin hbitos txicos inicia con cuadro de 8 das de evolucin acompaado de
fiebre y dolor centrotorcico intenso que aumenta con la inspiracin y los movimientos
respiratorios. En el ecocardiograma se objetiva un derrame pericrdico importante, sin
signos de compromiso hemodinmico. Cul sera su primer diagnstico?:

a)

Pericarditis aguda idioptica.

b)

Pericarditis tuberculosa.

c)

Taponamiento cardaco.

d)

Pericarditis de origen autoinmune.

La pericarditis aguda (PA) es un sndrome clnico debido a una inflamacin del pericardio
que se caracteriza por dolor torcico, roce pericrdico y alteraciones
electrocardiogrficas evolutivas.
Su incidencia y prevalencia son difciles de determinar. Estudios realizados en autopsias
muestran una prevalencia de alrededor de 1% en la poblacin general, lo que sugiere que la
presentacin en ocasiones se hace de manera subclnica. Representa un 5% de los dolores
torcicos no isqumicos que acuden a urgencias 1.
La causa ms frecuente de PA es la idioptica y/o viral, que corresponden a casi el 80% de
los casos, de hecho, los trminos idioptico y viral suelen utilizarse de forma indistinta 2-5.


Otras posibles causas son:
Infecciosa (7%):
Bacteriana: neumococo, estreptococo, estafilococo, neisseria, legionella.
Tuberculosa.
Vrica: Coxsackie, influenzae, VIH, hepatitis, adenovirus, echovirus.
Fngica: Histoplasmosis, coccidiomicosis, blastomicosis, candidiasis.
Otras: sfilis, protozoos, parsitos (entamoeba histolytica, toxoplasma).
Neoplasias (7%):
primarias: Mesotelioma, sarcoma, fibroma, lipoma
secundarias (metstasis): Pulmn, mama, linfoma, leucemia, carcinoides
La triada diagnstica clsica es: dolor torcico, roce pericrdico y alteraciones
electrocardiogrficas que suele ser precedida de fiebre, mal estado general y mialgias
(aunque en pacientes ancianos no suele presentarse la fiebre).
Los sntomas principales son:
Dolor torcico: dolor retroesternal o localizado en hemitrax izquierdo en la zona
precordial, de caractersticas pleurticas (aunque en ocasiones puede simular un cuadro
isqumico), que se puede irradiar al cuello o al trapecio izquierdo. Puede acompaarse de
sensacin de falta de aire. Ocasionalmente se localiza en regin epigstrica, simulando un
abdomen agudo. Suele aumentar con la inspiracin profunda, la tos, la deglucin y la posicin
supina y mejorar con la incorporacin a la posicin de sentado.
Roce pericrdico: es el hallazgo patognomnico de la exploracin fsica en la PA.
Corresponde al movimiento del corazn dentro del pericardio y se asemeja al ruido que
produce la friccin de cuero, por roce de las hojas pericrdicas inflamadas. Se da en
aproximadamente el 60-85% de los casos 7,8. Puede ser transitorio, monofsico, bifsico o
trifsico, segn la relacin que tenga con los movimientos cardiacos durante la sstole
auricular, sstole ventricular y el llenado ventricular rpido. Se ausculta mejor a nivel de
mesocardio y en parte baja de borde esternal izquierdo, sobretodo al final de la espiracin
con el paciente inclinado hacia delante. Es independiente de la existencia de derrame.
Cuando slo tiene un componente se puede confundir con un soplo sistlico mitral o
tricuspdeo. Es caracterstico del roce pericrdico su evanescencia (por lo que las
auscultaciones deben ser repetidas en varias ocasiones) y los cambios en sus
caractersticas segn la posicin en que se realice la exploracin.

Bibliografa:

LeWinter MM, Kabbani S. Pericardial diseases. En: Braunwalds heart disease. Douglas P.
Zipes editor. 7th. Ed. Philadelphia: Elsevier; 2005. p.1757-1780
Friman G, Fohlman J. The epidemiology of viral heart disease. Scand J Infect Dis Suppl
1993; 88: 710. [Medline]
Braunwald E. Enfermedades del pericardio. En: Harrison Principios de Medicina Interna.
Dennos L. Koper editores. 16 ed. Mxico: Mc-Graw-Hill; 2005. p. 1554-1571
Maisch B, Seferovic PM, Ristic AD, Erbel R, Rienmller R, Adler Y, et al, Grupo de Trabajo
para el Diagnstico y Tratamiento de las Enfermedades del Pericardio de la Sociedad
Europea de Cardiologa. Gua de Prctica Clnica para el diagnstico y tratamiento de las
enfermedades del pericardio. Versin resumida. Rev Esp Cardiol. 2004; 57:1090-114.
[Medline] [Texto completo]
Zayas R, Anguita M, Torres F, Gimenez D, Bergillos F, Ruiz M, Ciudad M, Gallardo A, Valles
F. Incidence of specific etiology and role of methods for specific etiologic diagnosis of
primary acute pericarditis. Am J Cardiol. 1995 Feb 15; 75(5):378-82.

41.- Ingresa en el servicio de urgencias un paciente que ha sufrido un grave accidente


automovilstico de frente. Se encuentra en un estado de agitacin, plido, ansioso,
hipotenso, con frialdad y discreta sudoracin fra de los miembros, presenta pulso
paradjico y respracin de kussmaul. La presin venosa se encuentra

aumentada.

El

diagnstico ms probable es:

a)

Su cuadro se debe a un shock hipovolmico.

b) Hay que descartar la existencia de un taponamiento cardaco.


c) Hay que examinar el abdomen y descartar que la causa de todo sea una rotura del bazo.
d) Fracturas costales con sncope vasovagal y gran ansiedad

El taponamiento cardaco se define como la compresin del corazn que resulta de la


acumulacin de lquido en el saco pericrdico y que produce un severo trastorno
hemodinmico.
El efecto principal de la compresin cardaca es una alteracin en el llenado de las
cavidades durante la distole, lo cual lleva a una disminucin del gasto cardaco y de la
presin arterial en un espectro variable en el que el paciente severamente comprometido
puede estar en shock cardiognico.
El taponamiento cardaco puede ser agudo o crnico, de origen traumtico o secundario a
casi cualquier tipo de pericarditis aguda o crnica, aunque es poco comn en la pericarditis
viral y despus del infarto agudo del miocardio. La causa ms frecuente de taponamiento
cardaco agudo es el hemopericardio causado por heridas con arma de fuego, arma blanca o
trauma cerrado de trax.
Las heridas punzantes y, especialmente, las lesiones por bala frecuentemente son fatales.
Sin embargo, si la herida penetrante es relativamente pequea suele haber taponamiento
cardaco, y el aumento de presin en el saco pericrdico puede ayudar a reducir la
severidad de la hemorragia.

El otro gran grupo de causas de taponamiento son las patologas mdicas, las cuales pueden
subdividirse en seis grupos: neoplsico, urmico, reumtico, hemorrgico, infeccioso e
idiomtico.
Diagnstico clnico
Las alteraciones fisiopatolgicas previamente mencionadas producen caractersticas
clnicas y hemodinmicas que ayudan al diagnstico. Con el aumento de la presin venosa se
detecta ingurgitacin yugular severa, la cual disminuye en forma visible durante la sstole,
por lo cual es posible encontrar la depresin X del pulso venoso magnificada sin que se
modifique la depresin Y (signo de Friedrich).
Con la acumulacin de lquido en el espacio pericrdico la intensidad de los ruidos cardacos
disminuyen y llegan en ocasiones a no ser audibles. La presin arterial se encuentra
generalmente disminuida y convergente (presin arterial diferencial menor de 30 mmHg) y
el paciente est taquicrdico.
El pulso paradjico es un signo casi constante en el taponamiento cardaco, lo mismo que el
signo de Kussmaul.
Estos hallazgos clnicos pueden corroborarse hemodinmicamente con un catter de
presin venosa central y una lnea arterial.
En el taponamiento cardaco agudo la historia de una herida en el trax, de un infarto
agudo del miocardio o de un aneurisma artico deben alertar sobre esta posibilidad, as
como el antecedente de neoplasia, insuficiencia renal o tratamiento anticoagulante
reciente.
Los pacientes generalmente aquejan dolor torcico sordo o constrictivo que puede ser
pleurtico. La disnea es frecuente y generalmente el paciente presenta sntomas asociados
con bajo gasto cardaco, como frialdad, debilidad muscular y diafresis, y puede en los
casos graves, exhibir alteracin neurolgica del estado de conciencia y shock cardiognico.
En el enfermo con taponamiento cardaco crnico los sntomas predominantes son los de la
enfermedad de base y se pueden encontrar prdida de peso, anorexia, debilidad marcada y
compromiso del estado general con disnea y dolor torcico crnicos. En estos pacientes es
frecuente encontrar signos de congestin venosa crnica como hepatomegalia, ascitis, y
edema de miembros inferiores.

LECTURAS RECOMENDADAS
Cuenca G. Taponamiento cardaco. En: Manual de Urgencias en Medicina Interna. Asociacin
Colombiana de Medicina Interna. Captulo Central.
Ediciones Acta Mdica Colombiana de Bogot, 1994.
Cheitlin MD, Abbott JA. Urgencias cardacas. En: Diagnstico y Tratamiento de Urgencias.
Editado por MT. Ho y CE Saunders.
Editotial El Manual Moderno. Mxico DF, 1991.

42.- Mujer de 23 aos G/1 la cual cursa con 5 semanas de gestacin tom cumarina hasta
el da en que se enter que estaba embarazada. Est preocupada de que la cumarina
ocasione defectos congnitos en el producto. En el caso anterior el defecto congnito ms
frecuente tras el uso de cumarina es:

a)
b)
c)
d)

Hipoplasia nasal
Espina bfida
Anencefalia
Acondroplasia

Tabla I
Frmacos cuya teratognia se ha comprobado en humanos

FRMACO

MALFORMACIONES DETECTADAS

RIESGO

ANTIBIOTICOS
Tetraciclinas

Tincin dental, hipoplasia enamel. Posible


retraso crecimiento seo.

Alrededor de 50% de
los expuestos a
tetraciclina; 12,5% de
expuestos a
oxitetraciclina.

Sndrome fetal de la warfarina: hipoplasia


nasal, condrodisplasia punctata,
braquidactilia, defectos craneales, orejas
anomalas, ojos malformados, malf. en
sistema nervioso central, microcefalia,
hidrocefalia, deformidades esquelticas,
retraso mental, atrofia ptica,
espasticidad; malformacin de Dandy
Walker.

El 16% tiene
malformaciones, un
3% hemorragias. Se
producen hasta un
8% de abortos.

Carbamacepina

Aumento del riesgo de sufrir defectos del


tubo neural.

Se estima un riesgo
del 1%.

Fenitoina

Sndrome fetal de la fenitoina: aplanammiento


puente nasal, pliegues epicantales internos,
ptosis, estrabismo, hipertelorismo, orejas
anormales o de baja implantacin, hipoplasia
de falanges distales y uas, anomalas
esquelticas, microcefalia y retraso mental,
deficiencias del crecimiento, neuroblastoma,
defectos cardacos, paladar hendido y labio

El 5-10% puede sufrir


el sndrome tpico. Un
30% parcialmente.
Hasta un 7% de
descendientes con
coeficiente
intelectual bajo.

ANTICOAGULANTES
Cumarinas

ANTIEPILEPTICOS


leporino.
Trimetadiona
(Troxidona)

Acido valproico

Sndrome fetal de la trimetadiona: retraso


crecimiento intrauterino, anomalas cardacas,
microcefalia, labio y paladar hendidos,
anomalas en orejas, facies dismorfica,
retraso mental, fstula traqueo-esofgica,
muerte postnatal.

Basndose en casos
notificado: 83% y
32% de muerte
infantil o neonatal.

Espina bfida con mielomeningocele, defectos


en sistema nervioso central, microcefalia,
defectos cardiacos.

Un 1% de riesgo de
defectos del tubo
neural.

Retraso del crecimiento, paladar hendido,


microftalmia, hipoplasia ovrica, opacidad
corneal, agenesia renal, malformaciones de los
dedos, defectos cardacos, otras anomalas
mltiples.

Entre el 10-50% de
los casos pueden
presentar
malformaciones,
aunque este riesgo,
extrado de series de
casos puede estar
exagerado.

Hidrocefalia, meningoencefalocele,
anencefalia, malf. craneales, hipoplasia
cerebral, retraso del crecimiento,
malformaciones oculares y del odo,
malformaciones nasales, paladar hendido,
malf. en miembros y dedos.

Entre el 7-75% de los


casos expuestos,
aunque este riesgo,
extrado de series de
casos puede estar
exagerado.

ANTINEOPLASICOS
Alquilantes
Busulfan
Clorambucil
Ciclofosfamida
Clormetina

Antimetabolitos
Aminopterina
Azauridina
Citarabina
Fluorouracilo
Mercaptopurina
Metotrexato

Sndrome fetal de la aminopterina: disostosis


craneal, hidrocefalia, hipertelorismo,
anomalas de odo externo, micrognatia,
paladar hendido.
HORMONAS
Dietilestilbestrol

Descendencia femenina:
Adenocarcinoma de clulas claras vaginal o
cervical en mujeres jvenes expuestas "in
utero" (antes de la semana 18); oligomenorrea,
reduccin de las tasas de embarazo,
incremento de las tasas de embarazos

Cambios morfolgicos
congnitos en epitelio
vaginal en el 39% de
las expuestas. En
exposicin antes de la
18 semana: riesgo de


pretermino, incremento de la mortalidad
perinatal y de aborto espontneo.

carcinoma 1.4 por


1000 de expuestas.

Descendencia masculina:
Quistes epididimarios, criptorquidia,
hipogonadismo, disminucin de la
espermatognesis, estenosis de meato,
hipospadias.
NEUROLEPTICOS
Litio

Mayor riesgo de anomala de Ebstein; no se ha


detectado un mayor riesgo de otro tipo de
malformaciones.

PENICILAMINA
Hiperclastosis cutnea (cutis laxa)

Pocos casos. Riesgo


desconocido.

Aborto espontneo, deformidades craneales,


de orejas, cara, corazn, extremidades e
hgado; hidrocefalia, microcefalia, defectos
cognitivos (incluso sin malformaciones
aparentes).

Isotretinoina: 38%. El
80% son
malformaciones del
sistema nervioso
central.

Focomelia, amelia, hipoplasia de miembros,


defectos cardacos congnitos,
malformaciones renales, criptorquidia,
parlisis VI par, sordera, microtia, anotia.

Alrededor del 20%


cuando la exposicin
ocurre entre 4-8
semana.

RETINOIDES
SISTEMICOS
Isotretinoina
Etetrinato

TALIDOMIDA

Bibliografa:

McBride WG. Thalidomide and congenital abnormalities. Lancet 1961; 2:1358.


Lenz W. Thalidomie and congenital abnormalities. Lancet 1962;1:45.
Shepard TH. Catalog of Teratogenic Agents. Johns Hopkins University Press
London, 1987.
Piper JM. Banur C, Kennedy DI. Prescription drugs use in pregnancy in a Medicaid
population. Am J Obstet Gynecol 1987; 157:148-156.
Salvador J, Martnez-Fras ML, Rodrguez Pinilla E. Consumo de medicamentos por
la mujer embarazada en Espaa. Ministerio de Sanidad y Consumo. Madrid, 1989.
Grupo de trabajo DUP. Estudio multicntrico sobre el uso de medicamentos durante
el embarazo en Espaa (I). Mtodos y caractersticas demogrficas de la poblacin
estudiada. Med Clin 1990;95:764-767.
Grupo de trabajo DUP. Estudio multicntrico sobre el uso de medicamentos durante
el embarazo en Espaa (II). Los frmacos utilizados durante la gestacin. Med Clin
1991;96:11-15.
Grupo de trabajo DUP. Estudio multicntrico sobre el uso de medicamentos durante
el embarazo en Espaa (III). Los frmacos utilizados durante el primer trimestre
de la gestacin. Med. Clin 1991;96:52-57.

Martnez-Fras ML. Medicamentos y teratogenia. Revisin bibliogrfica y situacin en


Espaa. Ministerio de Sanidad y Consumo. Madrid, 1989.

43.- Mujer de 37 aos. Acude a consulta por presentar cefalea, cansancio e irregularidades
menstruales con ritmo de 36 a 50 x 2-3 das. No se ha podido embarazar despus de 18
meses de actividad sexual regular. No tiene antecedentes importantes. EF: Campos visuales
normales, tiroides aumentada de tamao una vez y aumentada de consistencia, no tiene
galactorrea. Resto normal. Laboratorio: qumica sangunea, Bh y electrolitos normales.
Prolactina 47 ng/dL (< 25), perfil tiroideo: TSH 18 mUI/ml, T4t: 50 nmol/L (57.9 a 154.4),
T4L: 7.7 pmol/L (9 a 24), T3T: 1.06 nmol/L (1.2 a 2.9), T3L: 1.96 pmol/L (3 a 6.31)
La etiologa ms frecuente de este problema es:

a)
b)
c)
d)

Tiroiditis autoinmune crnica


Microadenoma hipofisario
Resistencia a la insulina
Enfermedad de Graves

Inicialmente el hipotiroidismo se diagnosticaba mediante la cuantificacin por tcnicas de


Radio Inmuno Anlisis (RIA) de las hormonas circulantes triyodotironina y tiroxina; el
proceso era lento y sometido a muchos factores de error que hacan su sensibilidad y
especificidad poco confiables. Posteriormente, se desarrollaron tcnicas para la medicin
de la TSH hipofisiaria igualmente mediante el RIA lo que mejor en forma importante la
sensibilidad para el diagnstico de esta enfermedad; sin embargo, los niveles de deteccin
de la prueba se encontraban en el orden de 1 IU/ml lo que haca que la prueba no fuera
sensible para valores menores de 1 IU/ml. Debido a esto se crearon tcnicas de segunda
generacin mediante la cuantificacin de TSH por anticuerpos monoclonales y RIA, el
IRMA (Immuno Radiometric with Monoclonal Antibodies) que permiti detectar valores de
TSH en rangos de 0.1 IU/ml; posibilitando desde entonces diagnosticar pacientes con
hipertiroidismo primario; pero con la limitante de que para esta tcnica era imposible
detectar valores de TSH menores de 0.1 IU/ml por lo que se creo la medicin de TSH
mediante quimioluminiscencia o mtodos enzimticos, es decir las tcnicas de tercera
generacin, las cuales pueden detectar valores de TSH de 0.01 IU/ml; con lo que se
logra el espectro ideal para una prueba de laboratorio que tiene la capacidad de
diagnosticar tanto la hipofuncin como la hiperfuncin(20).

Adems el avance no slo fue en la medicin de TSH sino tambin en las hormonas tiroideas
que han evolucionado simultneamente con la TSH y ya se miden incluso las fracciones
libres de hormonas y las fracciones totales, lo que ha facilitado el manejo de estos
pacientes. Gracias a esta evolucin en tcnicas de laboratorio, el diagnstico de
hipotiroidismo primario es bastante sencillo. Niveles de TSH superiores al valor mximo de
la tcnica seran diagnsticos de la disfuncin; pero no es tan fcil. Cuando tenemos un
paciente con toda la sintomatologa del hipotiroidismo y la TSH se encuentra elevada el
diagnstico es obvio; pero podemos tener pacientes con sntomas muy inespecficos como
depresin y con examen fsico normal a quienes se les encuentran valores de TSH por
encima del lmite superior y con hormonas tiroideas normales. Se trata de un hipotiroidismo
o es un valor ligeramente elevado ocasional de una persona sana (21). Igualmente tenemos
otra circunstancia que ha sido descrita con mayor frecuencia: pacientes con valores de
TSH en el lmite superior normal y con dislipidemia a quienes se les da tratamiento con
hormonas tiroideas y su dislipidemia se corrige manteniendo valores de TSH en rangos
normales. Todas las circunstancias anteriores han hecho que aparezca en el hipotiroidismo
primario la expresin de hipotiroidismo subclinico, que ha sido objeto de reuniones y
congresos dedicados exclusivamente a este tema. La sociedad Europea de Tiroides hace
algunas recomendaciones para el manejo de esta situacin que se consideran tiles como
gua (Tabla ).

Tabla. Enfoque del paciente con disfuncin tiroidea de acuerdo a los niveles de
hormona estimulante de la tiroides (TSH).
Si TSH
0.4 a 2.0
Si TSH < 0.4m U/L

mU/L

Si TSH > 5.0


Si TSH 2.01 a 5.0mU/L

mU/L

Normal,
Medir T3 y T4 totales o
libres para diagnostico
de hipertiroidismo.

Dar tratamiento

Repetir
cada cinco

Medir T4 libre y anticuerpos

para

aos

antitiroideos

hipotiroidismo

1. Si AAT (-) y T4 libre es


normal repetir screening cada
ao. Si TSH es > 4.0mU/l en dos
ocasiones dar tratamiento
2. Si AAT (+) y/o T4 libre esta
baja o normal baja tratar si
TSH es mayor de 3.0 mU/l y
observar a los otros

Tomado de Koutras DA. Subclinical hypothyroidism. En G. Hennemann, E.P. Krenning,


Thyroid International Merck KGaA, Darmstadt 1999 (3), 6-9

HIPOTIROIDISMO
PRIMARIO:
congnito: agenesia
autoinmune
postablativo
tiroiditis subaguda
deficiencia de yodo

SECUNDARIO
tumor hipofisiario
iatrognico

TERCIARIO

44.- Mujer de 22 aos que acude a consulta por prdida de peso, diarrea, palpitaciones,
temblor y labilidad emocional. No tiene antecedentes importantes. Refiere evacuaciones
diarreicas de 5 a 7 por da, sin moco o sangre.
EF: peso 52 kg, estatura 1.59, FC 108x, TA: 120/65, retraccin palpebral, hiperemia
conjuntival y de carncula, cuello con tiroides aumentada de tamao 3 veces de forma
difusa, extremidades superiores con hiperhidrosis palmar, temblor fino distal, reflejos
exaltados
El diagnstico probable es:
a)
b)
c)
d)

Parasitosis del tipo de la giardiasis.


Bocio multinodular txico.
Taquicardia paroxstica.
Enfermedad de Graves.

El bocio txico difuso (BTD) constituye la forma ms frecuente de hiperfuncin de la


glndula tiroidea (70 % de los casos), que puede aparecer a cualquier edad, aunque por lo
general aparece entre la tercera y cuarta dcada de la vida. Esta enfermedad es ms
frecuente en la mujer, donde se observa un predominio de 7:1 en relacin con los hombres
en regiones no bocigenas. Esta relacin se reduce en las zonas de bocio endmico. Los
factores genticos desempean un papel esencial en la etiologa y existe una predisposicin
familiar a esta enfermedad de
Graves-Basedow.
El BTD se caracteriza por la presencia de hipertiroidismo, bocio difuso y elstico,
oftalmopata, dermopata, acropaquia tiroidea y onicolisis. Es importante el diagnstico y
tratamiento precoz del hipertiroidismo para evitar complicaciones, principalmente las
cardiovasculares.
Criterios diagnsticos y teraputicos
Sospecha clnica
Sntomas de mayor valor diagnstico
Intolerancia al calor
Palpitaciones
Nerviosismo
Insomnio
Irritabilidad
Hiperquinesia
Fatigabilidad fcil
Polifagia
Perdida de peso
Hiperdefecacin y/o diarreas
Signos de mayor valor diagnstico
Bocio
Taquicardia
Temblor de las manos
Debilidad muscular
Piel hmeda y caliente
Hiperquinesia
Pelo fino y frgil
Bibliografa:
1. Bryer-Ash M. Evaluation of the patient with a suspected thyroid disorder. Obstet
Gynecol Clin North Am 2001
Jun;28(2):421-38
2. Weber AL, Randolph G, Aksoy FG. The thyroid and parathyroid glands. CT and MR
imaging and correlation
with pathology and clinical findings. Radiol Clin North Am 2000 Sep;38(5):1105-29


3. Jarlov AE, Nygaard B, Hegedus L, Hartling SG, Hansen JM. Observer variation in the
clinical and laboratory evaluation of patients with thyroid dysfunction and goiter. Thyroid
1998 May;8(5):393-8.

4. Costa AJ. Interpreting thyroid tests. Am Fam Physician 1995 Dec;52(8):2325-30

45.- Hombre de 50 aos, acude a consulta por hipercolesterolemia. Tiene antecedente de


infarto del miocardio hace 6 meses. Lleva dieta baja en colesterol. EF: peso 76 kg, estatura
170 cm, TA 120/80, FC 80x, resto de exploracin normal. Laboratorio: glucosa 76 mg/dl,
resto de qumica normal, colesterol 245 mg/dl, triglicridos 142 mg/dl, LDL 136 mg/dl,
HDL 36 mg/dl
El nivel de colesterol de LDL que debe tener el paciente es:
a)
b)
c)
d)

< 100 mg/dL


< 130 mg/dL
< 160 mg/dL
< 190 mg/dL

VALORES NORMALES
Parmetro
Colesterol
Colesterol-LDL

Colesterol-HDL
Triglicridos

mg/dl
< 200
< 100
< 130
<160
> 45 o >55
< 150

FACTORES DE RIESGO
CARDIOVASCULAR
Categorasque modifican las metas de LDL
Enfermedad coronaria y equivalentes <100
2 o ms factores de riesgo
<130
0 a 1 factor de riesgo
<160

46.- Hombre de 45 aos. Acude a la consulta solicitando un examen mdico. Tiene


antecedente familiar de diabetes en madre y dos tas maternas. El paciente es obeso desde
los 20 aos de edad. EF: peso 92 kg, estatura 1.75 m, TA 130/90, FC 78x, obesidad
generalizada. Laboratorio: glucosa 132 mg/dL, resto de qumica sangunea normal. Se
solicita otra glucemia que es de 129 mg/dL y una hemoglobina glucosilada con resultado de
7 %.
El diagnstico del paciente es:
a)
b)
c)
d)

Diabetes mellitus
Intolerancia a la glucosa
Tolerancia a la glucosa normal
Intolerancia a la glucosa de ayuno

47.- Un mdico, que no dispone de tablas de crecimiento y desarrollo, necesita valorar el


peso de un nio de 3 aos. Qu frmula entre las siguientes, debera utilizar para calcular
el peso medio de un nio normal de esa edad?
a)
b)
c)
d)

Edad en aos x 8 / 2
Edad en aos x 80 / 2
(Edad en aos x 2) + 8
Edad en meses x 10 / 2


Un nio sano duplica su peso a los 5 meses, lo triplica a los 12 meses y cuadriplica su peso a
los 2 aos (pesando 12 a 14kg). La respuesta C es la ms cercana al valor esperable, ya que
la talla al nacimiento es de 50cm, al los 12 meses: 50 + 50/2 = 75 cm; a los 24 meses: 50 +
50/2 + 50/4= 87cm; a los 4 aos: 2 x talla al nacimiento.

Nelson, Tratado de Pediatra 15 Ed, vol 1., pg. 35

48.- El mtodo ms frecuentemente usado para el tratamiento conservador de la


Displasia del Desarrollo de la cadera es:

a) Uso de triple paal


b) Uso de cojn de Frejka
c) Uso de arns de Pavlik
d) Uso de cojn de Creig

ARNES DE PAVLIK

La displasia en el desarrollo de la cadera es la principal indicacin para el uso del arns de


pavlik, teniendo como funcin el mantener la cadera centrada y de esta forma se permita
un mejor desarrollo de la misma

1- Dezateux C. Rosendahl K.Developmental dysplasia of the hip. Lancet 2007; 369: 154152.
2- Lee M., Craig P. Growth and Development of the Childs Hip. Orthop Clin N Am 37
(2006) 119 132.
3- U.S. Preventive services task force. Screening for Developmental Dysplasia of the Hip:
Recommendation Statement. American Family Physician Volume 73, Number 11, Junio 1,
2006.
4- Langman S. Embriologia Mdica con orientacion clinica. Decima edicion. Panamericana.
5- Rosselli P., Duplat J., Uribe i., Turriago C. Ortopedia Infantil. Editorial Panamericana.
6- Gelfer P, Kennedy K. Developmental Dysplasia of the Hip. J Pediatr Health Care. (2008).
22, 318-322.
7- Rachio KH. Simultaneuos open reduction and salter innominate osteotomya for
developmentaldysplasia of the hip. J Bone Joint Surg 2004;78B47:1-6.

49.- Una nia de 10 aos acude a consulta inicialmente con una historia de fiebre de 2 das
de evolucin y una adenopata cervical anterior izquierda fluctuante, sensible y caliente de
4 cm. cul es el diagnstico ms probable?


a)
b)
c)
d)

Enfermedad de Hodgkin
Leucemia linfoblstica aguda (LLA)
Histiocitosis X
Linfadenitis bacteriana aguda

La fiebre y los signos de supuracin sugieren infeccin bacteriana aguda. La enfermedad


de Hodgkin, la leucemia linfoblstica aguda, la histiocitosis X, y el neuroblastoma
metasttico pueden asociarse a fiebre y adenopatas, sin embargo, su aparicin no sera
tan aguda como se aprecia en la linfadenitis bacteriana aguda, y la supuracin difcilmente,
seria un signo de presentacin de ste problema.

50.- Una nia de 11 meses ingresa por tercera vez en un hospital con sibilancias sin fiebre
ni coriza. La madre dice que el primer episodio ocurri cuando tena 2 meses y que siempre
que ha ocurrido ha sido de forma brusca. Haba notado que el lactante

presento tos

nocturna antes del ingreso. Por otra parte, la nia tiene una causa ponderal ascendente y
recibe alimentacin con frmula enriquecida con protenas, y aunque babea mucho, este
problema parece estar resolvindose. En la familia no hay ninguna historia de fiebre del
heno, asma o eccema. En las dos ocasiones anteriores, la nia respondi parcialmente al
tratamiento con broncodilatadores, aunque haba seguido teniendo ligeras sibilancias entre
los episodios. Los hallazgos de la exploracin fsica son irrelevantes, a excepcin de roncus
bilaterales y sibilancias espiratorias. No existe sangre oculta en las heces, y el recuento
sanguneo completo no muestra eosinofilia perifrica. La radiografa de trax revela
hiperinsuflacin leve de ambos campos pulmonares, pero no infiltrados. Cul es la
explicacin ms probable para estas recadas de sibilancias?.

a) Reflujo gastroesofgico
b) Alergia alas protenas de leche de vaca


c) Fstula traqueoesofgica
d) Fibrosis qustica

Los rasgos tpicos del broncoespasmo por reflujo son: aparicin en edad temprana, inicio
brusco de las sibilancias sin elemento ambiental que lo desencadene, inexistencia de
historia de atopia familiar, tos nocturna que puede ser seal de reflujo con micro
aspiracin, respuesta escasa o incompleta a broncodilatadores. La tele de trax solo puede
mostrar cambios inespecficos de broncoespasmo. A los 22 meses de vida, ser de esperar
enfermedad parenquimatosa pulmonar, e infiltrados pulmonares en nios con fstula
traqueoesofagica as como fibrosis qustica.

51.- En un estudio de cohorte, los sujetos a estudiar deben cumplir con la siguiente
caracterstica:

a)
b)
c)
d)

Tener una historia familiar de la enfermedad en estudio


Que sean adultos con mltiples enfermedades a estudiar
No tener la enfermedad en estudio
Pertenecer a un grupo control

El estudio de cohorte es el diseo ideal para buscar la causa de una enfermedad por lo que
compara a un grupo de sujetos, libres de la enfermedad en estudio expuestos a los
factores de sospecha con otro grupo de sujetos tambin libres de la enfermedad, que no
estn expuestos a los factores sospechosos.

Ruiz M. A. Epidemiologa Clnica, 1. Ed. 2004; pg: 279,280.

52.- Para valorar que tan fuerte es la asociacin entre una exposicin y una enfermedad,
cul sera la medida de asociacin a calcular?
a) Riesgo relativo
b) Tasa de mortalidad
c) Incidencia


d) Prevalencia

El Riesgo Relativo mide la fortaleza de una asociacin entre un factor y un cierto resultado
final; de este modo, un Riesgo Relativo orienta hacia causalidad y es util para investigar el
origen de una enfermedad,

Morton R. F. Bioestadstica y Epidemiologa, Interamericana, 3. Ed. 1993; pg: 36

53.- Monte verde es una comunidad de 100,000 personas. Durante 2001 hubo 1,000
defunciones por todas las causas. Durante el mismo ao se registraron un total de 300
casos de Infartos Agudos al Miocardio y 60 defunciones por esta misma causa. La tasa de
mortalidad especfica para Infarto Agudo al Miocardio en Monte Verde es de:
a)
b)
c)
d)

20%
20 por 100,000
60 por 100, 000
10 por 100,000

La tasa de mortalidad para cualquier enfermedad especifica, puede expresarse para toda la
poblacin o para cualquier subgrupo de edad, raza o sexo. Se calcular dividiendo el nmero
de muertes causadas por la enfermedad especifica entre la poblacin total y se expresan
como muertes por 100, 000 habitantes por ao. ( TMe= 60/ 100, 000 por 100, 000)

Morton R. F. Bioestadstica y Epidemiologa, Interamericana, 3. Ed. 1993; pg: 22

54.- Femenino De 17 aos refiere que lleva un tiempo con menos apetito y durmiendo
menos ya que le cuesta poder dormirse. Es una buena estudiante, pero en la ltima
evaluacin le han quedado cuatro asignaturas y le cuesta concentrarse en los estudios. Los


dos ltimos fines de semana no ha salido con sus amigas porque no le apeteca y se siente
irritada con su familia, aunque no entiende porqu. Tiene dolores de cabeza y a veces le
viene la idea de la muerte a la imaginacin aunque piensa que no lo hara por sus
sentimientos religiosos. Nunca le haba sucedido algo parecido. El diagnstico sera:

a) Trastorno depresivo mayor.


b) Trastorno ciclotmico.
c) Anorexia.
d) Episodio depresivo mayor.

Criterios para el diagnstico del episodio depresivo mayor (DSM-IV)

A. Presencia de cinco (o ms) de los siguientes sntomas durante un perodo de 2 semanas,


que representan un cambio respecto a la actividad previa; uno de los sntomas debe ser (1)
estado de nimo depresivo o (2) prdida de inters o de la capacidad para el placer.
Nota: No incluir los sntomas que son claramente debidos a enfermedad mdica o las ideas
delirantes o alucinaciones no congruentes con el estado de nimo.

1. Estado de nimo depresivo la mayor parte del da, casi cada da segn lo indica el propio
sujeto (p. ej., se siente triste o vaco) o la observacin realizada por otros (p. ej., llanto).
Nota: En los nios y adolescentes el estado de nimo puede ser irritable
2. Disminucin acusada del inters o de la capacidad para el placer en todas o casi todas las
actividades, la mayor parte del da, casi cada da (segn refiere el propio sujeto u observan
los dems)
3. Prdida importante de peso sin hacer rgimen o aumento de peso (p. ej., un cambio de
ms del 5 % del peso corporal en 1 mes), o prdida o aumento del apetito casi cada da.
Nota: En nios hay que valorar el fracaso en lograr los aumentos de peso esperables.
4. Insomnio o hipersomnia casi cada da.


5. Agitacin o enlentecimiento psicomotores casi cada da (observable por los dems, no
meras sensaciones de inquietud o de estar enlentecido)
6. Fatiga o prdida de energa casi cada da.
7. Sentimientos de inutilidad o de culpa excesivos o inapropiados (que pueden ser
delirantes) casi cada da (no los simples autorreproches o culpabilidad por el hecho de
estar enfermo)
8. Disminucin de la capacidad para pensar o concentrarse, o indecisin, casi cada da (ya
sea una atribucin subjetiva o una observacin ajena)
9. Pensamientos recurrentes de muerte (no slo temor a la muerte), ideacin suicida
recurrente sin un plan especfico o una tentativa de suicidio o un plan especfico para
suicidarse.
B. Los sntomas no cumplen los criterios para un episodio mixto.

C. Los sntomas provocan malestar clnicamente significativo o deterioro social, laboral o de


otras reas importantes de la actividad del individuo.

D. Los sntomas no son debidos a los efectos fisiolgicos directos de una sustancia (p. ej.,
una droga, un medicamento) o una enfermedad mdica (p. ej., hipotiroidismo).

E. Los sntomas no se explican mejor por la presencia de un duelo (p. ej., despus de la
prdida de un ser querido), los sntomas persisten durante ms de 2 meses o se
caracterizan por una acusada incapacidad funcional, preocupaciones mrbidas de inutilidad,
ideacin suicida, sntomas psicticos o enlentecimiento psicomotor.

Bibliografa:
1. DSM-IV. American Psychiatric Association. . Diagnostic and Statistical Manual of Mental
Disorders (4th Ed.). Washington, DC.

55.- Se trata de masculino de 36 aos, trabaja como contador de una empresa textil, inicia
con comportamientos rituales de limpieza y actos repetitivos de comprobacin, con
arreglo a ciertas reglas que debe seguir estrictamente, se diagnostica finalmente
trastorno obsesivo-compulsivo Cul es el principal tratamiento psicofarmacolgico de
ste trastorno?

a) Antipsicticos.
b) Antidepresivos inhibidores de la recaptacin de noradrenalina.
c) Antidepresivos inhibidores de la recaptacin de serotonina.
d) Benzodiacepinas.

Tratamiento psicofarmacolgico
Los psicofrmacos han sido profusamente utilizados en el tratamiento del TOC. Durante un
largo perodo, de los aos 60 a los 90, el frmaco usado ha sido la clomipramina (Anafranil),
un antidepresivo tricclico al que tradicionalmente se relacion su eficacia con la reduccin
de la sintomatologa depresiva (Marks y cols., 1980). Al finalizar la dcada de los 80
aparecen un conjunto de nuevos frmacos, los inhibidores selectivos de la recaptacin de la
serotonina (ISRS), que, sustentados en el papel que la serotonina parece jugar en el TOC
(Barr, Goodman y Price, 1992), han supuesto un paso importante en el tratamiento
farmacolgico de este trastorno. La eficacia de los ISRS no parece ligada a la existencia
de sintomatologa depresiva, y adems tienen menos efectos secundarios que la
clomipramina (Rasmussen, Eisen y Pato, 1993, Freeman y cols., 1994).

56.- Se trata de femenino de 6 aos de edad el cual acude a consulta ya que ingiri 4
tabletas masticables de paracetamol 300 mg , para tomar medidas en caso de intoxicacin
Cul es la dosis txica oral de ste frmaco?

a) 10 mg/kg/dosis
b) 20 mg/kg/dosis
c) 40 mg/kg/dosis
d) 140 mg/kg/dosis.

La dosis oral txica en humanos est alrededor de los 10 g o 150 mg/kg.


Existe riesgo hepatotxico a partir de una ingesta de 15 g aunque se ha descrito alguna
muerte a dosis menores de 10g.
En los ltimos aos se est comprobando la aparicin de hepatotoxicidad ante dosis
teraputicas altas (6 g/da) en poblacin a riesgo.

Bromer MQ, Black M. Acetaminophen hepatotoxicity. Clin Liver Dis 2003;7:351-67.

57. En un paciente preescolar se debe sospechar el diagnstico de adenoiditis si ste


presenta:
a) Roncus,fiebre y dificultad respiratoria.
b) Tos nocturna, respiracin oral y voz nasal.
c) Rinorrea, conjuntivitis y disfona.
d) Halitosis, sibilancias y tos hmeda.

Signos y sntomas de adenoiditas:


Respiracin bucal y dificultad para la respiracin por la nariz.
Voz nasal.
Respiracin ruidosa.


Ronquidos durante el sueo.
Apnea (perodos en que deja de respirar durante unos segundos mientras duerme).
Resfriados frecuentes.
Tos nocturna.
Mordida Abierta.
Otitis Media Aguda y Sinusitis a repeticin.
Hiperactividad.
Dificultad para la concentracin.
Somnolencia diurna.

Nelson. Tratado de Pediatra. Tomo 1. 529-534. Ed. Interamericana.

58.- Le reportan larvas de Ancylostoma braziliensisi y Ancylostoma caninum son los


agentes causales en el hombre de:

a)
b)
c)
d)

Dermatitis de los nadadores.


Oncodermatitis.
Dermatitis verminosa reptante.
Larva currens cutnea.

Helmintos

Dermatitis verminosa reptante


Agente causal


Ancylostoma caninum, A. braziliense, Uncinaria stenocephala, Gnatostoma spinigerum, A.
tubaeforme, Bunostomum phlebotomum
Introduccin
La geohelmintiasis denominada larva migrans cutnea o dermatitis verminosa reptante
(DVR) es una entidad clnica que forma parte del complejo sndrome de las larvas
migratorias cutneas y viscerales.
Se define como una erupcin autolimitada de la piel producida por larvas de nemtodos
parsitos de diversos animales, cuyo husped natural no es el hombre, por lo que no pueden
completar en l su ciclo evolutivo.
Los agentes etiolgicos predominantes son A. caninum y A. braziliense. Se ha comprobado
que el primero puede alcanzar el tubo digestivo y producir el cuadro clnico conocido como
enteritis eosinoflica.

59.- Usted tiene la sospecha de diagnstico de criptosporidiosis, el examen de laboratorio


de eleccin para confirmar ste, es:
a)
b)
c)
d)

Coproparasitoscpico De Faust.
Coproparasitoscpico De Stoll.
Coproparasitoscpico Directo.
Frotis De Heces Teido Con Kinyoun.

Las aves infestadas con Cryptosporidium sp. no tienen historia, signos clnicos o lesiones
macroscpicas especficas Para el diagnstico de criptosporidiosis existen
aproximadamente 15 tcnicas para la observacin de oocistos de Cryptosporidium, pero las
ms usadas son las tcnicas de Kinyoun y Ziehl-Neelsen.
Jorge Tay Zavala, Parasitologa Mdica. Editorial: Mndez Editores, Edicin: 5 Ao: 1996.
Pginas: 81-96

60.- Femenino de 22 aos de edad, acude al servicio de urgencias por referir malestar
general, fosfenos, nusea y vmito. Antecedentes: cursa con embarazo de 36.5 SDG, G2,
A1, C0. Exploracin fsica: TA 185/110 mmHg, FC 120 lpm, FR 35 x, T37.9, alerta,
inquieta, aprecia ictericia, cardiopulmonar sin compromiso, abdomen globoso a expensas de
tero grvido, se detecta PUVI, longitudinal, ceflico, dorso a la derecha, FCF 142 x, al


tacto genital crvix, central, formado, cerrado, extraccin de guantes sin evidencia de
prdidas genitales, extremidades inferiores edema +++.
El dato de laboratorio que apoya el diagnstico en esta paciente es:

a)
b)
c)
d)

Hematuria.
Proteinuria.
Coluria.
Piuria.

Referencia:
La preeclampsia se define por aumento de la presin arterial y la presencia de proteinuria
durante el embarazo.
3) Leve: Las pacientes suelen tener unas cuantas manifestaciones, y su presin arterial
diastlica es menor de 110 mmHg. En ocasiones hay edema. La cifra de plaquetas es
mayor de 100 000/l.
4) Grave: Los sntomas son ms notorios y persistentes. La presin arterial casi
siempre es con niveles mayores de 160/110 mmHg. Puede haber trombocitopenia
que avance hasta coagulacin intravascular diseminada.

Se requiere hospitalizacin para las mujeres con preeclampsia; debe obtenerse una
biometra hemtica completa con cifra de plaquetas y determinacin de electrolitos, que
adems incluya enzimas hepticas. Se obtiene una muestra de orina de 24 horas para
determinar la depuracin de creatinina y protenas totales al ingreso hospitalario.
Se debe controlar la hipertensin arterial, para evitar sufrimiento fetal, as como
empeoramiento de la paciente.

Bibliografa:

1.-McPhee S, Papadakis M, et. al. Diagnstico Clnico y Tratamiento 2010. Lange, McGraw
Hill,


49 edicin, Mxico, 2010.
2.-Sibai BM, Diagnosis, prevention, and management of eclampsia. Obstet Gynecol. 2005.
Feb; 105;: 402 410.

61.- Al encontrarse realizando exploracin fsica por dolor abdominal, usted identifica
signo de Murphy positivo, esto se traduce en:

a) Dolor a la palpacin profunda en el punto cstico.


b) Dolor a la palpacin en el punto cstico durante la inspiracin.
c) Dolor a la palpacin profunda en el punto cstico durante la espiracin.
d) Dolor que interrumpe la inspiracin a la palpacin profunda en el punto cstico.

Maniobra de Murphy, el enfermo sentado y el examinador colocado atrs engancha el


punto cistico mientras el enfermo trata de realizar una inspiracin profunda, Es positiva en
la cocolecistitis aguda.

Apuntes de Semiologa
Semiologa mdica y tcnica exploratoria Escrito por Antonio Surs Batll,Juan Surs
Batll

62.- Cuando nos enfrentamos ante un cuadro clnico de amenorrea, galactorrea y prdida
de campo visual el diagnstico ms probable es:


a) Adenoma hipofisario no funcionante.
b) Intoxicacin por benzodiacepinas.
c) Prolactinoma.
d) Meningioma del tubrculo solar.

El prolactinoma es el tumor hipofisario ms frecuente, es de naturaleza benigna y pequeo


en 90 % de los casos. El cuadro clnico tpico en la mujer se compone de trastornos
menstruales, galactorrea y/o esterilidad; se acompaa de sntomas neurolgicos slo
cuando se extiende por arriba de la silla turca. Niveles de prolactina superiores a 100
ng/mL son prcticamente diagnsticos de prolactinoma, siempre y cuando no existan
embarazo y/o hipotiroidismo. La primera opcin teraputica del prolactinoma es la
farmacolgica con dopaminrgicos, lo que prcticamente ha eliminado la ciruga. Los
dopaminrgicos suprimen la sntesis y secrecin de prolactina con la consecuente
normalizacin del eje hipotlamo-gonadotrpico. Los dopaminrgicos son efectivos para
inducir la ovulacin y favorecen la consecucin de embarazo. Para cualquier dimensin del
prolactinoma se usan los dopaminrgicos durante uno a dos aos y generalmente despus de
suspenderlos se puede esperar que el tumor se reduzca de tamao y se corrija la
hiperprolactinemia. En contraste, en el hombre generalmente se encuentran
macroprolactinomas, mayores de 10 milmetros con extensin extraselar acompandose de
sntomas neurolgicos; sin embargo, tambin responden favorablemente a los
dopaminrgicos.

Gac Md Mx Vol. 140 No. 5, 2004

Referencias:
1. Schlechte JA. Prolactinoma. N Engl J Med 2003;349:2035-2041.


2. Zrate A, Canales ES, Jacobs LS, Soria J, Daughaday WH. Restoration of ovarian
function in patients with the amenorrhea-galactorrhea syndrome after long-term therapy
with L-Dopa. Fertil Steril 1973;24:340.
3. Tyson JE, Carter JN, Andreassen B, Huth J, Smith B. Nursing mediated prolactin
and luteinizing hormone secretion during puerperal lactation. Fertil Steril 1978;30:154.
4. Schlechte JA, Sherman BM, Chapler FK, VanGilder J. Long-term followup of women
with surgically treated prolactin-secreting pituitary tumors. J Clin Endocrinol Metab
1986;62:1296-301.
5. Losa M, Mortini P, Barzaghi R, Gioia L, Giovanelli M. Surgical treatment of
prolactin-secreting pituitary adenomas: early results and long-term outcome. J Clin
Endocrinol Metab 2002;87:3180-3186.
6. Zrate A, Canales ES, Cano C, Pilonieta CJ. Follow-up of patients with prolactinomas
after discontinuation of long-term therapy with bromocriptine. Acta Endocrinol
1983;104:139-42.
7. Zrate A, Canales ES, Alger M. The effect of pregnancy and lactation on pituitary
prolactin secreting tumors. Acta Endocrinol 1979;92:407-11.
8. Bevan JS, Webster J, Hburke J, Scanlon MF. Dopamine agonists and pituitary tumor
shrinkage. Endocr Rev 1992;13:220-240.

63.- - Una mujer de 17 aos tiene exantema cutneo rojo difuso; fiebre de 39.4C y
diarrea leve acuosa. En fechas recientes tuvo infeccin de garganta por la que se le
administr sulfametoxazol. Comenz sus menstruaciones hace tres das. En la exploracin
fsica se encuentran cambios eritematosos difusos de la piel con descamacin temprana. La
boca y las conjuntivas estn eritematosas. Cul de los siguientes explica todo el proceso?

a) Bacteriemia por Salmonella


b) Sndrome de choque txico (TSS)
c) Tuberculosis
d) Mononucleosis por virus de Epstein-Barr

Allen R. M. MMS Medicina Interna. 5. Edicin. National Medical Series. Mc. Graw Hill.
2006. (Captulo 8 VII D 1 a, 2 b). La bacteriemia por Salmonella, el sndrome de choque
txico (TSS), la tuberculosis y la mononucleosis de Epstein-Barr pueden acompaarse de
fiebre, pero la presencia de exantema descamativo difuso sugiere TSS, reaccin
farmacolgica grave (p. ej., sndrome de Stevens-John-son); enfermedad de Kawasaki, o
escarlatina. El exantema cutneo relacionado con salmonelosis es muy sutil y evanescente
(manchas de color rosa). La tuberculosis no se caracteriza por afeccin cutnea difusa y de
mucosas o diarrea acuosa. Si bien la alergia al sulfametoxazol puede producir eritema
cutneo y de mucosas, no causa diarrea.

64.- Masculino de 4 aos de edad con antecedentes de otitis media a los doce meses de
edad, actualmente cursa con otalgia, otorrea
general. A la exploracin

e hipoacusia, acompaado de malestar

en conducto izquierdo se aprecia

Hiperemia timpnica,


hipervascularizacin y engrosamiento timpnico. El tratamiento de eleccin en esta
patologa es:

a) Antihistamnicos de forma exclusiva


b) Solo medidas generales
c) Drenaje, AINES y esteroides locales
d) Antibiticos, antihistamnicos y AINES.

Otitis media que se acompaa con secrecin y signos locales, sistmicos, o ambos.
Instituto Nacional de la Nutricin. Salvador Zubiran Manual de teraputica mdica y
procedimientos

de

urgencias.

Cuarta

edicin.

Pag.

443

450.

Mac

Graw-Hill

Interamericana. Mxico. La otitis media se define como la inflamacin del odo medio que
se acompaa de secrecin y signos locales, sistmicos, o ambos, de enfermedad aguda. Esta
enfermedad predomina en la infancia, sin embargo tambin existe en el adulto. Su
distribucin tiene una clara periodicidad estacional, sobre todo en otoo e invierno. Los
agentes de mayor prevalencia en nuestro pas son: Streptococcus pneumoniae, Haemophilus

influenzae, Morexella catarrhalis estreptococo del grupo A y S. aureuse. Los antibiticos


ms aceptados en estos casos son la amoxicilina, la amoxicilina con clavulanato, la
calritromicina y el trimetropim con slfametoxazol. Se acepta el uso de antihistamnicos y
antiinflamatorios no esteroideos para disminuir la congestin de la mucosa para resolver la
obstruccin de la trompa farngotimpnica; en realidad no se ha demostrado efecto sobre
la duracin de los sntomas. El drenaje del derrame solo esta indicado cuando ste persiste
por ms de tres semanas.

65.- El riesgo de adquirir infeccin por virus del papiloma humano entre estudiantes
universitarios se mide en un estudio epidemiolgico a travs de:


a) La incidencia acumulada
b) La densidad de incidencia
c) La tasa de incidencia dividida entre la prevalencia
d) Prevalencia

La incidencia de una enfermedad se mide de dos formas: mediante la densidad de incidencia


que expresa la ocurrencia de la enfermedad entre la poblacin en relacin con unidades de
tiempo-persona, por lo que mide la velocidad de ocurrencia de la enfermedad y la incidencia
acumulada que expresa el volumen de casos nuevos ocurridos en una poblacin durante un
periodo, y mide la probabilidad o riesgo de los miembros de una poblacin, de contraer una
enfermedad en un periodo especfico.

Moreno A. Principales medidas en epidemiologa. Rev Salud Pblica Mex, 2000;42(4): 343

66.- Un hombre de 55 aos se le diagnostica HPB, se niega a tomar tratamiento


farmacolgico y elige una RTUP, cul es la complicacin ms frecuente de este
procedimiento:

a)
b)
c)
d)

Contractura de cuello vesical


Impotencia
Incontinencia
Eyaculacin retrgrada


Reseccin Transuretral (RTU): es el gold standard de las intervenciones.
__ En general es una ciruja corta.
__ 90% mejora importante de los sntomas.
__ Morbimortalidad baja del 0,2%.
__ el 20 % debe reoperarse a los 10 aos aproximadamente.
__ complicaciones: impotencia 10%, eyaculacin retrograda 50%,
incontinencia 4%.

BIBLIOGRAFIA:
1. Medina JJ, Parra RO, Moore RG. Benign prostatic hyperplasia (the aging prostate). Med
Clin North Am 1999 Sep;83(5):1213-29.
2. Oesterling JE. Benign prostatic hyperplasia. Medical and minimally invasive treatment
options. N Engl J Med 1995 Jan 12;332(2):99-109.

67.- Masculino de 69 aos, el cual ingresa al servicio de urgencias quejndose de dolor


torcico de 3 horas de evolucin, con diaforesis, palidez. El EKG muestra elevacin del
segmento ST en la cara inferior. Cual de las siguientes es una contraindicacin absoluta
para trombolisis:

a) Enfermedad convulsiva crnica


b) Enfermedad de Parkinson temprana sin demencia
c) Antecedente de evento vascular cerebral hemorrgico
d) Historia de traumatismo hace 1 ao en accidente vehicular


CONTRAINDICACIONES A LA TROMBOLISIS
Absolutas

Relativas

Antecedente de AVE hemorrgico

- Tratamiento anticoagulante.

- Aneurisma disecante.

- Hipertensin arterial refractaria > 180

- Ditesis hemorrgica.

/110 mmHg.

- Hemorragia digestiva en el mes

- Maniobras de resucitacin cardiopulmonar

precedente.

prolongadas.

- Ciruga o traumatismo reciente

- Embarazo.

(ltimas 3 semanas).

- Puncin de vaso en sitio no compresible


(vena subclavia).
- AVE isqumico en los ltimos 6 meses.

Como se explic previamente, a mayor precocidad de terapia, se logra una mayor reduccin
de la mortalidad. Por ejemplo, si logramos reperfundir a un paciente con menos de 3 horas
de evolucin, la mortalidad cae en un 50% en comparacin con la terapia realizada en un
paciente con 12 horas de evolucin, en el cual la mortalidad cae slo en un 10%. Otro factor
que influye en la disminucin de la mortalidad en los pacientes tratados con trombolisis es
la extensin del infarto, ya que a mayor isquemia hay mayor disminucin de mortalidad.
BIBLIOGRAFIA
1. Overview of the management of suspected myocardial infarction. Reeder G.S., Kennedy
H.S., Rosenson R.S. UpToDate v9.1, 2001.
2. Diagnosis of acute myocardial infarction with biomarkers of cardiac injury. Jaffe Allan
S. UpToDate v9.1, 2001.
3. Braunwald: Heart Disease: A Textbook of Cardiovascular Medicine, 6th ed., 1114-1207.
W. B. Saunders Company, 2001.
4. Rakel: Conn's Current Therapy 2001, 53rd ed., 335-348, W. B. Saunders Company, 2001
5. Rosen: Emergency Medicine: Concepts and Clinical Practice, 4th ed., 1655-1979. MosbyYear Book, Inc., 1998.
6. Futterman L.G., Lemberg L. Update on management of acute myocardial infarction:
facilitated percutaneous coronary intervention. American journal of critical, 2000;9:70-6.
7. Wright R.S., Kopecky S.L., Reeder G.S. Update on intravenous fibrinolytic therapy for
acute myocardial infarction. Mayo Clinic proceedings, 2000;75:1185-91.


68.- Masculino de 12 aos que acude a consulta por dolor anterior de rodilla. El dolor es
desencadenado o intensificado por el ejercicio fsico intenso, al subir y bajar escaleras y
arrodillarse. A la exploracin fsica presenta una prominencia excesiva de la tuberosidad
tibial con dolor a la palpacin, no presenta derrame articular y los arcos de movilidad estn
respetados. El diagnstico ms probable es:

a) Enfermedad de Friesberg
b) Enfermedad de Kholer
c) Enfermedad de Sever
d) Enfermedad de Osgood-Schlater

La enfermedad de Osgood-Schlatter u osteocondrosis y en otros casos Epifisitis es una


hinchazn dolorosa de la protuberancia en la parte frontal y superior de la tibia, llamada
espina tibial anterior que afecta a nios y adolescentes que estn teniendo un crecimiento
acelerado mientras practican un deporte.
Se cree que esta enfermedad es causada por lesiones pequeas y usualmente inadvertidas,
ocasionadas por la sobrecarga repetitiva antes de que se complete el crecimiento del rea.
Tambin se cree de otro factor que pueda afectar la tibia que marca como culpable a la
tirantez de los msculos que se encuentran anteriores al cudriceps que est unido con los
tendones que corren por la rodilla hasta la tibia para conectar ambos msculos. Al momento
de contrarse el tendn puede a comenzarse a jalar el hueso de la pantorrilla causando el
dolor. Este trastorno se observa muy a menudo en adolescentes activos y atlticos,
generalmente en edades comprendidas entre los 10 y los 15 aos. Es comn en adolescentes
que juegan ftbol, bsquetbol y voleibol, al igual que aquellos que practican gimnasia

1.

Osgood R.B. (1903). Lesions of the tibia tubercle occurring during adolescence.
Boston Medical and Surgical Journal 148: pp. 1147.
2. Schlatter C. (1903). Verletzungen des schnabelfrmigen Forsatzes der oberen
Tibiaepiphyse. [Bruns] Beitrge zur klinischen Chirurgie 38: pp. 87487.
3. Nowinski RJ, Mehlman CT (1998). Hyphenated history: Osgood-Schlatter disease.
Am J. Orthop. 27 (8): pp. 5845.

69.- Masculino recin nacido que presenta una conjuntivitis neonatal, para indicar el
tratamiento usted relaciona los siguientes grmenes por ser los de mayor frecuencia
causantes de esta enfermedad:

a)
b)
c)
d)

Clamidia, gonococo, estafilococo aureus


Clamidia, treponema, gonococo
Estreptococo grupo A, listeria
Listeria, pseudomonas

Las conjuntivitis neonatales (CN) siguen siendo uno de los motivos ms frecuentes de
consulta, pudiendo llegar a comprometer seriamente la capacidad visual del beb,
conduciendo incluso a la ceguera. La incidencia de esta enfermedad oscila entre 1,6%-12%
en el primer mes de vida.

Las CN pueden ser de origen infeccioso o qumico. Las CN de origen infeccioso pueden ser
adquiridas durante la gestacin, durante el parto por contacto con la secrecin uretrovaginal, o bien en el post-parto y el mbito de convivencia diaria. Las CN que se manifiestan
entre las 24 y las 48 horas del nacimiento son de origen qumico, debidas a la profilaxis
efectuada por instilacin de solucin de nitrato de plata en el saco conjuntival del neonato,
para prevenir la proliferacin de Neisseria gonorrhoeae .

Se debe tener presente la adquisicin de la infeccin perinatal a partir del ambiente en que
se halla el beb. La etiologa ms frecuente en este caso corresponde a Staphylococcus

aureus,

Haemophilus

influenzae

Streptococcus

pneumoniae).

En las ltimas dcadas, Chlamydia trachomatis ha resultado ser el microorganismo


prevalente entre los agentes causales de enfermedades de transmisin sexual, con una
prevalencia del 2 al 35% en embarazadas. La transmisin vertical se da entre el 60 y 70%
en los hijos de madres infectadas .El riesgo de adquirir conjuntivitis en estos bebs es del
18 al 60%, mientras que el de neumona es del 10 al 25%.

En el caso de N. gonorrhoeae , la infeccin ocular puede complicar el cuadro con ceguera y


artritis.
Desde el ao 1995, en nuestro hospital se lleva a cabo en forma rutinaria la bsqueda de
grmenes comunes (GC) y C. trachomatis en todo recin nacido con conjuntivitis.

Bibliografa
Krachmer, Manis, Holland. Cornea and External Disease: Clinical Diagnosis and
Management, Vol II. Mosby, 1997.
Yanoff, Ophtalmology. Mosby, 1999.

70. - Female patient 23, who will have an operation with a history of hepatitis. Which of
the following anesthetics are contraindicated in your case?

a) Halothane
b) Lidocaine
c) Fentanyl
d) Ketamine

El halotano reduce el flujo sanguneo esplcnico y heptico como consecuencia de

una

reduccin de la presin de riego. Puede producir necrosis heptica fulminate, que se


caracteriza por fiebre, anorexia, nusea y vmito que pueden durar varios das despus de
la anestesia y acompaarse por un exantema y eosinofilia perifrica. Este sndrome recibe
el nombre de hepatitis por halotano.
Hardman J, Limbird L. Goodman and Gilman Las bases farmacolgicas de la teraputica. Mc
Graw Hill 2002. 361.

71.- Femenino de

34 aos

que inicia con debilidad muscular, episodios frecuentes de

ptosis, diplopia y fatiga generalizada. Refiere dificultad a la masticacin y deglucin. La


prueba ms especfica para corroborar su diagnstico es:

a) Electromiograma de fibra muscular aislada.


b) Determinacin de anticuerpos anti-receptor de acetilcolina.
c) TAC torcico.
d) Electromiograma con estimulacin repetitiva.

La miastenia gravis es una enfermedad autoinmune que se caracteriza por presentar


debilidad muscular fluctuante y fatiga de distintos grupos musculares. La miastenia gravis
afecta a individuos de todas las edades, con una predileccin por mujeres entre los 20 y 40
aos. Los msculos oculares, faciales y bulbares son los ms frecuentemente afectados por
la enfermedad. Los pacientes con miastenia gravis manifiestan empeoramiento de la
debilidad muscular, con infecciones intercurrentes, fiebre y agotamiento fsico o
emocional. La infeccin respiratoria (bacteriana o vrica) es la causa ms frecuente de
provocacin. La presencia de anticuerpos contra receptores de acetilcolina en un paciente
con manifestaciones clnicas compatibles con la miastenia gravis confirma el diagnstico. El
tratamiento de esta entidad es controvertido y debe ser individualizado, ya que no existe
un rgimen teraputico uniforme para todos los pacientes. Entre los tratamientos
disponibles

testacan

los

frmacos

anticolinestersicos,

los

corticosteroides,

la

plasmafresis, la inmunoglobulina, los inmunosupresores y la timectoma.

REFERENCIAS
1. Drachman DB. Myasthenia gravis. N Engl J Med 1994;330:1797-8102.2. Engel AG. Myasthenic syndromes. In Engel AG, Franzini-Armstrong C, eds. Myology. 2
ed. New York: McGraw-Hill;1994. P. 1798-835.
3. Engel AG. Ohno K. Milone M, Sine SM. Congenital myasthenic syndromes caused by
mutations in acetylcholine receptor genes. Neurology 1997;48 (Suppl 5):S28-35.
4. Steinman L, Mantegazza R. Prospects for specific inmunotherapy in myasthenia gravis .
FASEB J 1990;4:2726-31.
5. Massey JM. Treatment of acquired myasthenia gravis. Neurology 1997;48 (suppl 5):S4651.

72.-

Una mujer de 40 aos con DM acude a consulta quejndose de descarga vaginal

prurtica y blanquecina, cual de los siguientes exmenes de diagnsticos es mas til para
identificar el patgeno?

a) Wet prep.


b) Tincion de Gram
c) PH
d) KOH

Prueba del KOH es un procedimiento en el cual hidrxido del potasio (KOH) se utiliza
detectar hongos disolviendo las clulas humanas en una cultura. La diferencia en la
composicin de la pared de clula de clulas humanas y de clulas fungicidas permite que
este procedimiento ayude a distinguir las dos clulas. El KOH desnaturaliza las protenas en
la clula humana; solamente sigue habiendo las clulas fungicidas ser considerado debajo
del microscopio.
1. Bernal B. Fisiologa y ecologa de la vagina. Rev Chil Obstet Ginecol 1986; 51:56-60.
2. Ibrcena E. Vaginosis bacteriana; diagnstico y prevalencia. XII Congreso Peruano de
Obstetricia y Ginecologa 1996; 204-6.
3. Scapini JC, Guzmn CA. Deteccin de bacilos Gram negativos curvos anaerobios en
pacientes con vaginosis. Obstet Ginecol Latinoam 1986; 44: 320-5.
4. Soihet S. El flujo vaginal en la consulta ginecolgica. Ginecol Obstet (Per) 1986; 30: 5060.
5. Linaldi CAS, Urbina JR, Castaeda JL. Vaginitis por Gardnerella vaginalis en nias y
adolescentes. Bol Med Infant Mex 1988; 45: 101-3.
6. Sobel J. Vaginitis and vaginal flora. Controversies abound. Current Opin Infect Dis 1996;
9: 42-7.
7. Faro S. Leucorrea. Causas infecciosas e imbalances en el ecosistema vaginal. Tribuna
Mdica. Cahners Healthcare ed 1996; 1 10.

73.- Cul de los siguientes hallazgos es el ms indicativo de sndrome nefrtico agudo, en


el anlisis de orina?

a) Lipiduria.
b) Cilindros hemticos.
c) Cilindros granulosos.
d) Cilindros hialinos.

El sndrome nefrtico (SN) es un conjunto de enfermedades caracterizadas por inflamacin


de los glomrulos renales con el consecuente deterioro de su funcin. La inflamacin es por
lo general inmune, aunque puede resultar ser de origen infeccioso.1 Como resultado aparece
una prdida sbita de sangre (hematuria) y de protenas en la orina (proteinuria) y una
cada rpida del ndice de filtrado glomerular (VFG).

El sndrome nefrtico tiene mltiples causas, entre ellas causas de origen infeccioso que
afectan al glomrulo, cuyo origen puede ser propiamente renal tanto como sistmico.
Adems hay causas que involucran la autoinmunidad, trastornos metablicos y traumatismos
entre otros.
En jvenes y adolescentes
Las enfermedades causales que se observan con ms frecuencia en nios y adolescentes
son, entre otras:

Nefropata por IgA.


Prpura de Schnlein-Henoch.
Sndrome urmico hemoltico.
Glomerulonefritis pos-estreptoccica, el prototipo de la enfermedad nefrtica
aguda.

El motivo de consulta de un paciente con sndrome nefrtico, por lo general, es referir


edema, orinas oscuras, micciones de bajo volumen y poco frecuentes y, tardiamente,
aparecen dolor lumbar, dificultad respiratoria y convulsiones.2 El examen fsico puede
mostrar signos de retencin nitrogenada e hipertensin arterial.
Hematuria
En el 75% de los pacientes con un sndrome nefrtico se presenta una hematuria
macroscpica y es uno de los sellos clnicos de los pacientes con este sndrome.7
Caractersticas de la hematuria de origen glomerular:

Color oscuro
Total (presente en el primer, segundo y tercer chorro miccional)

Indolora
Sin cogulos
Al examen microscpicos, los hemates no se observan frescos.

Presencia de Cilindros Hemticos, lo cual es casi patognomnico de la hematuria


glomerular.

Estos datos son caractersticos de la hematuria glomerular a la vez que la hematuria


glomerular es caracterstica del sndrome nefrtico.
1.
2.

3.
4.

5.
6.

(agosto de 2007). Sndrome nefrtico agudo (en espaol). Enciclopedia mdica


en espaol. Consultado el 14 de febrero, 2008.
Tierney, Lawrence M., McPhee, Stephen J., and Papadakis, Maxine A. Current:
Medical Diagnosis and Treatment, 2003 (en ingls). Publicado por McGraw-Hill
Professional Publishing, 2002; edicin 42; pg 886. ISBN 0-07-139593-8
Alan Stevens, James Lowe ANATOMIA PATOLOGICA (en espaol). Publicado por
Elsevier Espaa, 2001; pg 357. ISBN 84-8174-512-X
Jimenez-Murillo, Luis; A. Berlango Jimnez y F. J. Montero Prez (2004). Medicina
de urgencias, 3ra edicin (en espaol), Elsevier, Espaa, pp. 493 ISBN
848174672X.
Kumar, Vinay; Ramzi S. Cotran y Stanley L. Robbins (2003). Patologa humana, 7ma
edicin (en espaol), Elsevier, Espaa, pp. 522 ISBN 8481746665.
Segn Goic. Semiologa Mdica. Santiago de Chile: Editorial Mediterrneo. (1990) p.
255-256.

74.- Ante un paciente masculino de 25 aos de edad que presenta hematuria macroscpica
al da siguiente de una infeccin farngea y edemas maleolares. Dos aos antes haba
presentado un cuadro similar que desapareci con rapidez, por lo que no haba consultado
previamente. En la exploracin fsica existe una T.A. de 170.100mmhg, Creatinina en suero
de 2.2 mgrs/dl Cul de los siguientes es el diagnstico ms probable?:
a) Glomerulonefritis post-estreptoccica aguda.
b) Glomerulonefritis membrano-proliferativa.
c) Hialinosis Segmentaria y Focal.
d) Glomerulonefritis rpidamente progresiva.

Se caracteriza por la proliferacin de las clulas del mesangio y un engrosamiento de las


paredes capilares del glomrulo. Se denomina tambin mesangiocapilar o
hipocomplementmica. Habitualmente, es idioptica o primaria y se clasifica en tipo I
(depsitos subendoteliales) y tipo II (depsitos densos intramembranosos). Afecta a los


nios y a los adultos jvenes y, en ms del 50% de los casos, se manifiesta como sndrome
nefrtico. Es habitual la disminucin de C3 y la presencia del C3 nephritic factor.
Evoluciona hacia la insuficiencia renal y es muy frecuente la recidiva postrasplante en el
tipo II.

Donadio J. Membranoproliferative glomerulonephritis. In:


R. Schrier and C. Gottschalk: Diseases of the Kidney.ln: 4th
Edition. Boston: Little Brown, 1998: 2035-60.
Schmitt H, Cavalcanti de Oliveira J, Bohle A: Tubulo-intersticiai
aiterations in type 1 membranoprolrterative
glomerulonephritis. Pathol Res Pract 1997; 182:6-l 0.
Burkholder PM. Atlas of Human Glomerular Pathology
Hagerstown: Harper and Row, 1974; 189.
Kim Y, Michael A, Fish A. Idiopathic membranoproliferative
glomerulonephritis. In: Brenner BM and Stein JH: Nephrotic
Syndrome. New York: Churchill Livingstone, 1982: 237-58.

75.- Acude al servicio paciente masculino de 66 aos de edad el cual refiere disminucin
de la visin en ambos ojos con varios meses de evolucin, esta disminucin se presenta a la
visin de lejos y ms acentuada en la visin prxima o de lectura. Adems ha notado mayor
prdida visual con luz solar intensa y se deslumbra con mayor facilidad. No refiere
alteraciones en la percepcin de los colores, aunque s cree verlos ms apagados, tampoco
refiere metamorfopsias. Cul de los siguientes diagnsticos le parece ms compatible con
el cuadro descrito?

a) Aumento de la presbicia.
b) Desarrollo de cataratas.
c) Desarrollo de neuropata ptica anterior isqumica.
d) Desarrollo de glaucoma.

Cataratas.
Cualquier opacidad del cristalino, conlleve o no incapacidad funcional.
CATARATA CONGNITA.
Por alteraciones a partir de la 4 5 semana de embarazo. Aparecen al nacer o en los
primeros tres meses de vida. Provocan ms del 10% de las cegueras en edad escolar.
Pueden ser:
Hereditarias (10-25%). Bien aisladas o asociadas a malformaciones oculares o sistmicas.
Comportamiento familiar y generalmente bilaterales.
Embriopatas. Por infecciones intrauterinas, sobre todo durante el primer trimestre de
embarazo, como rubola, toxoplasmosis y citomegalovirus.
Metablicas.
- Galactosemia. Por dficit del enzima galactosa 1-P uridil transferasa, que se transforma
en galactitol. Este se deposita en el cristalino, provocando una retencin de agua que lo
opacifica. A medida que el nio va tomando leche van apareciendo cataratas,
hepatoesplenomegalia y retraso mental. Reversible, si se diagnostica en las primeras fases
de la enfermedad.
- Hipoparatiroidismo o pseudohipoparatiroidismo: cuando el Ca2+ es menor de 9,5 mg/100
ml, se puede originar catarata.
Txicas. Por clorpromacina, corticoides, hipervitaminosis D.
Carenciales. Por dficit de vitamina A, triptfano, cido flico o vitamina B12.
Cromosomopatas. Sndrome de Down (50% tienen cataratas),
Turner.

CATARATAS SECUNDARIAS.
Aparecen en el curso de procesos oculares o sistmicos.
Oculares: queratitis, uvetis, tumores coroideos, traumatismos, desprendimiento de retina
y glaucoma, entre otras.
Sistmicas:
Metablicas. En diabetes, hipertiroidismo, enfermedad de Wilson, distrofia miotnica de
Steinert

Sindermatticas. En patologa dermatolgica: poiquilodermia, esclerodermia y eccema


atpico.
Txicas. Por metales como talio, plata, mercurio, hierro, cobre, o frmacos como
corticoides, miticos, antimitticos, ergotamina.
CATARATA SENIL.
Forma ms frecuente de catarata y causa ms frecuente de prdida visual reversible en
pases desarrollados. Son bilaterales, aunque de desarrollo asimtrico.
CLNICA.
Se produce una disminucin progresiva de la agudeza visual, sin dolor ni inflamacin. Mejora
en ambientes poco iluminados o tras instilar un midritico, y empeora en ambientes muy
iluminados. En algunos pacientes los sntomas comienzan con una mejora de la presbicia,
por un aumento en el ndice de refraccin del cristalino. Tambin pueden aparecer
fotofobia, visin de halos coloreados y diplopa monocular (debidos a diferencias de
refraccin entre zonas de la lente).
Puede ser nuclear, cortical (por hiperhidratacin) o subcapsular posterior. Se denomina
incipiente cuando la opacidad y el trastorno visual son mnimos, madura cuando la opacidad
es total y el dficit visual severo, e hipermadura, cuando se produce disolucin y


licuefaccin de las fibras, perdiendo su estructura y apareciendo de color gris homogneo.
La cpsula puede presentar pliegues y el ncleo caer hacia abajo.
MUY IMPORTANTE
En un paciente de edad avanzada que progresivamente desarrolla disminucin de la agudeza
visual que empeora con luz intensa y mejora de la presbicia, debemos sospechar catarata
senil.

COMPLICACIONES.
Si la catarata no se opera, pueden aparecer las siguientes complicaciones:
Iridociclitis. Por salida de las protenas del cristalino a travs de la cpsula.
Glaucoma secundario agudo o facomrfico. El cristalino capta agua y aumenta de volumen,
provocando un aplanamiento de la cmara anterior y un cierre angular con aumento de PIO.
Glaucoma facoltico. Protenas cristalinianas que pasan a cmara anterior y provocan una
obstruccin de la malla trabecular.
Luxacin del cristalino.
DIAGNSTICO.
El diagnstico general de cataratas se hace provocando midriasis farmacolgica y
observando a simple vista, apareciendo una leucocoria en los estadios avanzados; o bien
observando mediante oftalmoscopia directa a unos 30 cm, aprecindose manchas oscuras
sobre el reflejo rojo del fondo del ojo, o impidiendo ver este reflejo cuando son maduras; o
bien con lmpara de hendidura, que permite ubicar la opacidad dentro del cristalino.
TRATAMIENTO.
El tratamiento es exclusivamente quirrgico. La tcnica ms utilizada se denomina
facoemulsificacin

76.- Femenino de 50 aos de edad, originaria de Granada, Espaa y radicada en la Ciudad


de Mxico refiere rubor, tumefaccin y rigidez de las articulaciones interfalngicas
dstales desde hace tres meses, pero no presenta otras molestias articulares. Cul de los
siguientes diagnsticos es ms probable?


a) Osteoartritis erosiva.
b) Artritis reumatoide.
c) Espondilitis anquilosante.
d) Esclerodermia.

Allen R. M. MMS Medicina Interna. 5. Edicin. National Medical Series. Mc. Graw Hill.
2006. (captulo 10 V E 1). Tpicamente, la osteoartritis erosiva afecta las articulaciones
interfalngicas dstales en mujeres de edad madura. Es improbable que esos sntomas
articulares dstales prominentes sucedan en pacientes con artritis reumatoide o con lupus
eritematoso diseminado sin molestias articulares ms generalizadas. No hay pruebas que
indiquen espondilitis anquilosante o esclerodermia.

77.- Acude una paciente al servicio de urgencias, por referir dolor y enrojecimiento del
ojo derecho, con el antecedente de eritema nodoso. Se valora por oftalmologa y es
diagnosticada de uvetis anterior. Su placa de trax muestra adenopatas hiliares
bilaterales. A que patologa nos estamos enfrentando?

a) Infeccin postestreptoccica
b) Sarcoidosis
b) Tuberculosis pulmonar
d) Sfilis. .

Sarcoidosis
La sarcoidosis, (del griego sarx, que significa "carne") o enfermedad de Besnier-Boeck, es
una enfermedad granulomatosa sistmica, de caracter autoinmune, que afecta a todas las
poblaciones y etnias humanas, y fundamentalmente a adultos entre 20 y 40 aos. Sus
causas son desconocidas. Los sntomas pueden aparecer repentinamente, pero suelen
manifestarse de manera gradual. El curso de la enfermedad es variable y puede ser desde


asintomtica hasta crnica (en el 5% de los casos), pudiendo llevar a la muerte (slo en un
1% de los casos en que se hace crnica).
La enfermedad puede manifestarse en cualquier rgano del cuerpo, con mayor frecuencia
en el pulmn y ganglios intratorcicos. Tambin son frecuentes las manifestaciones
oculares, cutneas y hepticas.

Sintomatologa
La sarcoidosis es una enfermedad sistmica que puede afectar a cualquier rgano. Los
sntomas ms comunes son imprecisos: fatiga, falta de energa, prdida de peso, dolores
articulares, sequedad ocular, visin borrosa, falta de aliento, tos seca o lesiones cutneas,
que varan desde enrojecimientos y pequeos ndulos hasta eritema nodoso o lupus. A
menudo es asintomtica.
La combinacin de eritema nodoso, linfadenopata bilateral y dolor articular recibe el
nombre de sndrome de Lfgren, y tiene un pronstico relativamente bueno.
Pueden producirse afecciones renales, hepticas, cardiacas o enceflicas, que pueden
provocar alteraciones funcionales posteriores. Las manifestaciones oculares incluyen
uveitis, uveoparotitis (asociada con el sndrome de Heerfordt-Waldenstrom) e inflamacin
de retina, que puede provocar prdida de visin clara o ceguera.
Las manifestaciones oftalmolgicas de la sarcoidosis son variadas. Cuando las lesiones
aparecen slo en el fondo de ojo sin otra sintomatologa asociada, se corre el riesgo de
detectar tardamente la enfermedad sistmica. As, ante el diagnstico dudoso de
sarcoidosis se debe realizar una exploracin minuciosa del fondo de ojo, con el fin de
identificar lesiones sospechosas. Adems es importante observar la existencia de estas
lesiones, ya que un 37% de ellas se asocian a las alteraciones en el sistema nerviosos
central, producidas por la propia sarcoidosis (1).
La vasculitis retiniana es la alteracin ms frecuente asociada a sarcoidosis que se observa
en el fondo de ojo.
Las lesiones granulomatosas coroideas son raras, usualmente se presentan con poca
sintomatologa y ceden bien con tratamiento esteroideo, dejando solamente alteraciones,
por disrupcin en el epitelio pigmentario de la retina.


Su importancia tambin radica en el diagnstico diferencial que hay que realizar con los
tumores coroideos, siendo el carcinoma metastsico y el melanoma amelantico los ms
frecuentemente confundidos (2).
Chumley LC, Kearns TP: Retinopathy of sarcoidosis. Am J Ophthalmol 1972; 73:
123.
2. Letocha CE, Shields JA, Goldberg RE: Retinal changes in sarcoidosis. Can J

1.

Ophthalmol 1975; 10: 184.

3. Baughman RP, Lower EE, du Bois RM. Sarcoidosis. The Lancet


2003/3/29;361(9363):1111-8.
4. Nunes H, Bouvry D, Soler P, Valeyre D (2007). Sarcoidosis Orphanet J Rare

Dis. Vol. 2. pp. 46. PMC 2169207. DOI 10.1186/1750-1172-2-46. PMID 18021432.
5. Henke, C. E., G. Henke, L. R. Elveback, C. M. Beard, D. J. Ballard and L. T. Kurland.
1986. The epidemiology of sarcoidosis in Rochester, Minnesota: a population-based
study of incidence and survival. Am. J. Epidemiol. 123:840845.
6. "American Thoracic Society: Statement on Sarcoidosis. Am J Respir Crit Care Med
1999;160:736-755.

78. A una mujer de 30 aos se le diagnostica una neuritis ptica retrobulbar. Qu enfermedad sistmica debe sospecharse por su mayor frecuencia de asociacin a esta
patologa oftalmolgica?
El diagnstico oftalmolgico de neuritis ptica retrobulbar en paciente femenino, est
ligado estrechamente a la siguiente patologa sistmica:

a) Esclerosis mltiple.
b) Espondilitis anquilopoytica.
c) Artritis reumatoide.
d) Diabetes mellitus.


La esclerosis mltiple (EM) es una enfermedad desmielinizante, neurodegenerativa y
crnica del sistema nervioso central. No existe cura y las causas exactas son desconocidas.
Puede presentar una serie de sntomas que aparecen en brotes o que progresan lentamente
a lo largo del tiempo. Se cree que en su gnesis actan mecanismos autoinmunes.
Se distinguen varios subtipos de esclerosis mltiple y muchos afectados presentan formas
diferentes de la enfermedad con el paso del tiempo.

A causa de sus efectos sobre el sistema nervioso central, puede tener como consecuencia
una movilidad reducida e invalidez en los casos ms severos. Quince aos tras la aparicin
de los primeros sntomas, si no es tratada, al menos 50% de los pacientes conservan un
elevado grado de movilidad. Menos del 10% de los enfermos mueren a causa de las
consecuencias de la esclerosis mltiple o de sus complicaciones.
Es, tras la epilepsia, la enfermedad neurolgica ms frecuente entre los adultos jvenes
(desde la erradicacin de la poliomielitis) y la causa ms frecuente de parlisis en los pases
occidentales. Afecta aproximadamente a 1 de cada 1000 personas, en particular a las
mujeres. Se presenta cuando los pacientes tienen entre 20 y 40 aos.
Los sntomas de la esclerosis mltiple pueden ser leves o severos, de larga o de corta
duracin, y pueden aparecer en distintas combinaciones, segn el rea del sistema nervioso
afectada. Una remisin completa o parcial de los sntomas, especialmente en las etapas
iniciales de la enfermedad, ocurre en un 70 por ciento, aproximadamente, de los pacientes
con esclerosis mltiple.
El sntoma inicial de la esclerosis mltiple es, a menudo, visin borrosa o doble,
distorsin del color rojo-verde o incluso ceguera en un ojo. Inexplicablemente, los
problemas visuales tienden a desaparecer en las etapas posteriores de la esclerosis
mltiple. Los problemas inflamatorios del nervio ptico pueden diagnosticarse como

neuritis retrobulbar o neuritis ptica. Cincuenta y cinco por ciento de los pacientes
con esclerosis mltiple tendrn un ataque de neuritis ptica en algn momento de su vida y
ste ser el primer sntoma de la esclerosis mltiple en un 15 por ciento aproximadamente,


de los casos. Esto ha llevado al reconocimiento general de la neuritis ptica como un
sntoma inicial de la esclerosis mltiple, especialmente si las pruebas diagnsticas tambin
revelan anormalidades en el lquido de la mdula espinal del paciente.

Neuritis ptica bulbar o papilitis. La papila ptica est elevada y de bordes borrosos.

Bibliografa:
American Academy of Ophthalmology, Basic and Clinical Science Course: NeuroOphthalmology, AAO 2004-2005.
Burde, R, Savino PJ, Trobe JD, Clinical decisions in Neuro-Ophthalmology, 3rd edition,
Mosby, 2002, St Louis.
Kline, Lanning, Bajanadas, Frank J, Neuro-ophthalmology review manual, 5th edition, 2004.
Lee, Andrew, Brazis, Paul, Clinical Pathways in Neuro-Ophthalmology, 2nd edition, Thieme,
2003.
Martin, Timothy, Corbett, James, Neurooftalmologa, 1a edicin, Ediciones Harcourt, 2001,
Madrid.
Miller, Neil, Newman, Nancy, Walsh & Hoyts Clinical Neuro-Ophthalmology The Essentials,
5th edition, Lippincott, Williams & Wilkins, 1999.


Miller, Neil, Newman, Nancy (editores), Walsh & Hoyts Clinical Neuro-Ophthalmology, 6th
edition, Lippincott Williams & Wilkins, 2004.
Pea, Luis, Manual de Oftalmologa, Mediterrneo, Santiago de Chile, 2002.
Snell, RS, Neuroanatoma clnica, 5a edicin, Editorial mdica Panamericana, 2001.

79.- Una mujer de 80 aos de edad con antecedente de insuficiencia cardiaca congestiva
presenta angina de pecho. Sus medicamentos se ajustan con: furosemida, digoxina,
nitroglicerina y potasio complementario. Poco despus presenta cefaleas pulsatiles
intermitentes. Cul de las siguientes medidas debe tomar primero el mdico?

a) Comenzar la administracin de propranolol


b) Comenzar la administracin de ergotamina sublingual
c) Realizar una tomografia computadorizada del cerebro
d) Suspender la nitroglicerina

Allen R. M. MMS Medicina Interna. 5. Edicin. National Medical Series. Mc. Graw Hill.
2006. (captulo 11IV B 1 a (2), d (1) (a), 4 a-b). La nitroglicerina puede causar cefaleas
"vasculares" terebrantes; por tanto, la ms sencilla opcin de tratamiento es suspender el
preparado de nitroglicerina y utilizar, si es posible, un medicamento cardaco alternativo.
Siempre debe considerarse la arteritis temporal como posible causa de cefalea en
pacientes mayores de 50 aos. Favorecen este diagnstico el aumento de la velocidad de
eritrosedimentacin; la claudicacin mandibular; las artralgias y las mialgias; y una arteria
temporal hipersensible e indurada. El propranolol es un antimigraoso eficaz. Sin embargo,
antes de prescribir estos frmacos deben descartarse los posibles factores que la
precipitan. La ergotamina es un tratamiento abortivo eficaz para migraa, pero es un
vasoconstrictor y no debe usarse en pacientes con angina de pecho. En la valoracin de
ancianos con cefalea de inicio reciente debe considerarse realizar una tomografa
computadorizada del cerebro. En este caso, si es posible suprimir la cefalea al suspender la
nitroglicerina, no es necesaria una CT.

80.- Un empleado de hospital de 27 aos de edad presenta induracin en el sitio de prueba


con derivado de protena purificado (PPD) realizado como parte de un examen de muestreo
sistemtico. Cul de los siguientes datos est en contra de la quimioprofilaxis con
isoniazida?

a) Induracin de 4 mm de dimetro.
b) Cutirreaccin con PPD negativa un ao antes.
c) Cutirreaccin positiva un ao antes.
d) Exposicin extensa reciente a un vecino con tuberculosis activa.

29. La respuesta es A.- Allen R. M. MMS Medicina Interna. 5. Edicin. National Medical
Series. Mc. Graw Hill. 2006. (captulo 8 VII C 3). Una induracin de 4 mm de dimetro en el
sitio de prueba con derivado de protena purificado (PPD) va contra el uso de
quimioprofilaxis con isoniazida. A todos los adultos menores de 35 aos con prueba PPD
positiva se les realiza quimioprofilaxis con isoniazida a no ser que no puedan tolerarla y ya
hayan recibido tratamiento completo, o se sepa que han estado expuestos a tuberculosis
resistente a isoniazida. Se requiere una induracin de por lo menos 10 mm para considerar
una prueba positiva. Los grados menores de induracin pueden representar reacciones
cruzadas a otras micobacterias.


81.- Masculino de 49 aos de edad quien se refiere a s mismo como sano y sin molestias.
Acude a una revisin general por indicacin de la empresa en la que labora. En sus estudios
de laboratorio se encontr un aumento moderado de

TSH (15mU/ml) con T3 y T4

normales. La primera posibilidad diagnstica es:

a)
b)
c)
d)

Hipertiroidismo.
Hipotiroidismo subclnico.
Mixedema.
Tiroiditis por anablicos.

Halabe J. Mercado M. Nellen H. Tiroides. Gua prctica para el clnico Manual Moderno.
Mxico. Primera Edicin. Pag. 42. Mencin aparte merece el caso del aumento leve a
moderado de TSH (5 a 20 mU/mL), con hormonas tiroideas normales, lo que generalmente
ocurre en un paciente asintomtico y se conoce como hipotiroidismo subclnico. Sin
embargo,

es

posible

encontrar

en

estos

pacientes

fatiga,

piel

seca,

sntomas

neuromusculares o intolerancia al fro, con mayor frecuencia que en personas sanas del
mismo sexo y edad. En ocasiones hay bocio o incluso oftalmopata distiroidea.

82.- Acude a su consulta paciente femenina de 26 aos que cursa el tercer trimestre del
embarazo tiene edema en miembros inferiores sin sintomatologa agregada, usted indica a
la paciente:

a) Elevacin de los miembros inferiores en decbito lateral


b) Restriccin Hdrica
c) Tiacidas
d) Diurtico de asa


DeCherney A. (1999) Diagnstico y tratamiento ginecoobsttricos. Mxico. Ed. Manual
Moderno. Pag 237. El edema de las partes bajas producido por la impedancia del retorno
venoso es comn en la parte final del embarazo, La paciente debe tratarse solo si est
molesta. La elevacin de los miembros inferiores (especialmente en decbito lateral)
mejora la circulacin. Estn contraindicados los diurticos.

83.- Femenino de 19 aos, soltera, nulpara con VSA desde los 17 aos, acude a consulta por
referir desde hace 3 meses presenta hemorragia irregular o postcoital, no refiere otra
sintomatologa, usted debe sospechar cervicitis por:

a) Chlamydia
b) Micoplasma
c) Gardnerella
d) Candida

Berek J. (2002) Ginecologa de NOVAK. Mxico. Ed. Mc Graw Hill Interamericana. Pag 293.
La hemorragia irregular o poscoital puede relacionarse con cervicitis por Chlamydia. La
adolescentes tienen las tasas ms altas de infecciones por Chlamydia que cualquier grupo
de edad, y deben investigarse estos microorganismos de manera sistemtica entre las
adolescentes sexualmente activas.

84.- Paciente masculino de 10 aos de edad que inici su padecimiento hace 24 hrs al
presentar, posterior a la ingesta de pescado, evacuaciones diarreicas, dolor abdominal,


nusea y vmito; acompandose de cefalea, parestesias periorales y bradicardia. El da de
hoy se agreg la presencia de ataxia y refiere que se quema al tocar el agua fra. Cul es
el agente etiolgico ms probable?

a) Vibrio cholerae
b) Agente Norwalk
c) Ciguatera

d) StaphIlococcus aeurus

U.S. Food & Drug Administration - Center for Food Safety & Applied Nutrition Foodborne Pathogenic Microorganisms and Natural Toxins Handbook. La intoxicacin por
ciguatera se produce por ingerir pescados como dorado, barracuda, cubera, etc.
contaminados con macroalgas con ciguatoxina-1. la evolucin de la intoxicacin es la
siguiente: en las primeras 12 horas se presentan manifestaciones gastrointestinales como
dolor abdominal, nusea, vmito y diarrea; posteriormente se agregan datos neurolgicos
como calambres, parestesias, mialgias, fatiga, ataxia, y disgeusia y finalmente alteraciones
cardiovasculares (bradicardia, taquicardia o hipotensin.) Los datos principales son las
parestesias periorales y la inversin de la sensacin trmica, es decir sentir fro al tocar
objetos calientes y viceversa. Adems se puede presentar ceguera temporal, parlisis de
los msculos faciales, oftalmoplejia, espasticidad, delirio, sensacin de prdida de los
dientes, exacerbacin del acn, sialorrea, cada del cabello, uas y descamacin de las piel.

85.- Paciente de 40 aos con ndulo mamario indoloro, de bordes imprecisos. La


mamografa revela imagen nodular, con espculas en todos sus mrgenes, y 10
microcalcificaciones finas, agrupadas en el interior. El diagnstico ms probable, entre los
que se citan, es:

a) Fibroadenoma.
b) Carcinoma


c) Quiste
d) Mamografa normal para la edad de la paciente.

Tcnicas diagnsticas
Exploracin
Masa palpable o engrosamiento unilateral. La posibilidad de que una masa palpable en la
mama sea maligna est en relacin con mayor edad, postmenopausia y con las siguientes
caractersticas en el examen fsico: consistencia firme, aspecto slido, bordes irregulares,
escaso desplazamiento sobre la piel, la regin costal o los tejidos que le rodean, unilateral,
no dolorosa y la presencia de adenopatas axilares. Sin embargo, an en ausencia de estos
factores un 10% pueden ser malignas, algunas veces una zona de engrosamiento que no llega
a masa puede ser cncer.

La coexistencia de masa y adenopata axilar palpable debe considerarse cncer mientras no


se demuestre lo contrario. El 90 % de las masas suelen ser lesiones benignas. Las masas de
superficie lisa y consistencia elstica estn asociadas a fibroadenoma en mujeres entre
20-30 aos y a quistes en las mujeres de 30 a 40. La exploracin a realizar ante esta
situacin es una mamografa si hay antecedentes de cncer de mama y una ecografa sobre
todo si existe dolor (ICSI, 2005).
Secrecin por el pezn. Siempre se debe estudiar. Hay mayor riesgo de lesin maligna en el
caso de que la secrecin contenga restos hemticos y est asociado a masa. La citologa del
lquido expulsado slo puede ser tenida en cuenta si es positiva. Est indicado realizar
mamografa y galactografa en el caso de que el exudado se presente en un solo conducto.
La presencia de secrecin lechosa bilateral orienta a causa endocrinolgica se ha de
realizar el diagnstico diferencial de galactorrea (ICSI, 2005).
Dolor. Es uno de los motivos de consulta mas frecuente. En ausencia de masa otros
sntomas de sospecha suele ser debida a tensin premenstrual, dolor condrocostal y a otras
causas (ICSI, 2005). Est asociado con mayor frecuencia a cambios fibroqusticos en la
mama premenopusica.
Sntomas cutneos. La Enfermedad de Paget afecta al pezn y areola de forma unilateral,
clnicamente muy similar a la dermatitis crnica crnica eccematosa se asocia a un
carcinoma mamario intraductal subyacente. (Fitzpatrick, 2001)


La retraccin del pezn o de la piel de presentacin reciente se debe evaluar
cuidadosamente. Los fenmenos inflamatorios del tipo de eritema, induracin, aumento de
temperatura y dolor pueden ser indicativos de un tumor inflamatorio de mal pronstico. En
ocasiones un tumor evolucionado puede dar lugar a un cncer ulcerado.

Imgenes
Mamografa
Tiene una sensibilidad y especificidad del 90%, siendo el mtodo aislado de diagnstico ms
eficaz, aunque en mamas densas pierde sensibilidad. Utilizada para el screening puede
reducir la mortalidad del cncer de mama en un 33%. Nos puede dar el diagnostico, la
presencia de multicentricidad o de lesiones sincrnicas.
La mamografa nos va a valorar distintos tipos de imgenes:

1: Signos primarios:
A.

Masa dominante: Valorando tamao, densidad, forma nitidez y estabilidad en el

seguimiento. Es la lesin ms frecuentemente hallada.


Considerando la clnica y los datos mamogrficos, ecogrficos, etc. se establece la
probabilidad de malignidad del ndulo, pudiendo expresarla en las siguientes cuatro
categoras:
o

Benigno

Probablemente benigno

Probablemente maligno

Maligno

NDULO BENIGNO

NDULO PROBABLEMENTE BENIGNO

NDULO PROBABLEMENTE MALIGNO

NDULO MALIGNO

B.

B.Lesiones estrelladas (o de alteracin de la arquitectura): Representadas por reas de


distorsin de la arquitectura mamaria, de bordes irregulares y que adoptan una morfologa
radiada. Suele ser un signo temprano en el carcinoma de mama y es de difcil
interpretacin.

Posee

menos

valor

predictivo

positivo

que

el

ndulo

las

microcalcificaciones, por lo que se recomienda biopsia quirrgica en todos los casos


excepto en los que los antecedentes de traumatismo, ciruga previa o inflamacin permitan
optar por el seguimiento de la lesin. En estos casos siempre es conveniente haber
realizado una citologa con resultado negativo.
C.

Microcalcificaciones: Son hallazgos frecuentes y el anlisis de sus caractersticas

nos puede ayudar a diferenciar las benignas de las sospechosas y de las claramente
malignas.
Las benignas no requieren mas pruebas diagnsticas complementarias, las probablemente
benignas precisas de un seguimiento mamogrfico no inferior a los dos aos y en las


sugestivas de malignidad, la biopsia es preceptiva. Hay que analizar las siguientes
caractersticas:

Tamao: Las superiores a 2 mms. se clasifican de macrocalcificaciones y suelen ser

benignas. Por debajo de los 2 mms. Se denominan microcalcificaciones y cuanto ms


pequeas y agrupadas ms sospechosas son de malignidad

Morfologa: Las calcificaciones malignas suelen ser heterogneas en forma y

tamao, puntiagudas, anguladas, irregulares, en "coma", ramificadas y con forma de punto y


raya. Las benignas suelen ser homogneas, redondas y en ocasiones anulares y de centro
claro.

Nmero: Se considera que cuando hay cinco o ms calcificaciones menores de 1 mm.

en un rea de 1x1 cm. de mamografa, existe sospecha de malignidad. Cuanto mayor es el n


de calcificaciones en esa rea, ms sospechosas son.

Distribucin: Las calcificaciones distribuidas de forma segmentaria, no al azar, son

sospechosas e indicativas de biopsia.

Variacin en el tiempo de las calcificaciones: Las calcificaciones malignas varan con

el tiempo. La estabilidad de las calcificaciones durante ao y medio - dos aos, se


consideran como benignas.

Calcificaciones asociadas a mama: Los carcinomas de mama calcifican en un 50 %.

Cuando hay calcificaciones internas en lesiones con signos de malignidad, aumentan las
posibilidades de malignidad. Se hallan en un 75% de los cnceres ocultos y suponen el 3047% de hallazgo aislado en los cnceres de mama. Son el primer marcador de cncer de
mama en las mujeres jvenes.

2: Signos secundarios:
1.

Engrosamiento de la piel.

2.

Permeabilidad linftica.

3.

Aumento de la vascularizacin.

4.

Afectacin linftica.

5.

Dilatacin ductal.

Bibliografa:
Apantaku LM. Breast cancer diagnosis and screening. [Internet]. American Family
Physician;

2000

[acceso

28/6/2007].

Disponible

en:

http://www.aafp.org/afp/20000801/596.html.
Barratt A, Howard K, Irwig L, Salkeld G and Houssami N. Model of outcomes of
screening mammography: information to support informed choices. BMJ 2005;330;936940.[Texto completo]
Brewer NT, Salz T, Lillie SE. Systematic review: the long-term effects of falsepositive mammograms. Ann Intern Med. 2007;146(7):502-10 [PubMed] [Texto completo]
Breast Imaging reporting and data system (BI-RADS). 2nd ed. Reston (VA):
American College of Radiology; 2007 [Resea]
Cantin J, Scart H, Levine M, Hugi M. Clinical practice guidelines for the care and
treatment of breast cancer: 13. Sentinel lymph node biopsy. Can. Med. Assoc. J. 2001; 165:
166 173 [Texto completo]
Dynamed. Breast cancer (female9) [Internet]. Ebsco industries ;2007 [acceso
3/7/22007]. Disponible en http://dynamed102.ebscohost.com/Detail.aspx?id=114433
Eberl MM, Fox ChH, Edge SB, Carter CA, Mahoney MC. BI-RADS Classification for
Management of abnormal Mammograms. J Am Board Fam Med 2006;19:161-4 [PubMed]
[Texto completo]

Fitzpatrick TB, Johnson RA, Wolff K, Suurmod D. Atlas color y sinopsis de


dermatologa clnica. Madrid: McGraw Hill Interamericana; 2001. p.494-5

86.- Se trata de recin nacido pretermino de 30 semanas de gestacin, peso de 1200 gr.
Curs con

enfermedad de membranas hialinas por lo que se manejo con una dosis de

surfactante y manejo con ventilacin convencional por 3 das. Posteriormente inici con


alimentacin enteral por sonda orogstrica con leche humana, sin tolerancia a la misma
(residuo gstrico y distensin abdominal).
La primera sospecha diagnstica es:

a)
b)
c)
d)

Alergia a las protenas de la leche


Alergia a la soya
Enterocolitis necrotizante
Atresia intestinal

El sndrome clnico ha sido clasificado en estadios por Bell y col. (1978) y modificado
por Walsh y Klegman (1986) para incluir hallazgos sistmicos, intestinales y
radiolgicos.

A. Estadio I : sospecha de enterocolitis necrotizante

Los hallazgos sistmicos son inespecficos.

Los hallazgos intestinales incluye el residuo gstrico y heces guayaco positivas.

Los hallazgos radiolgicos son normales e inespecficos.

B. Estadio II A: enterocolitis necrotizante leve

Los hallazgos sistmicos son similares al estadio I.

Los hallazgos intestinales incluyen distensin abdominal prominente con


hipersensibilidad a la palpacin o sin ella, ruidos hidroareos ausentes, sangre
macroscpica en materia fecal.

Los hallazgos radiolgicos, leo con asas dilatadas con reas focales de neumatosis
intestinal.

C. Estadio II B: enterocolitis necrotizante moderada

Los hallazgos sistmicos incluyen acidosis leve y trombocitopenia

Los hallazgos intestinales incluyen edema de la pared abdominal e hipersensibilidad


a la palpacin con una masa palpable o sin ella.

Los hallazgos radiolgicos incluyen neumatosis extensa y ascitis temprana.

Puede haber gas en la vena porta intraheptica.

D. Estadio IIIA: enterocolitis necrotizante avanzada:

Los hallazgos sistmicos incluyen acidosis respiratoria y metablica, ventilacin


asistida por apnea, hipotensin arterial, oliguria, neutropenia y coagulacin intravascular
diseminada.

Los hallazgos intestinales incluyen edema que disemina, eritema e induracin del
abdomen.

Los hallazgos radiolgicos incluyen ascitis prominente y asa centinela persistente


sin perforacin.
E. Estadio IIIB: enterocolitis necrotizante avanzada:

Los hallazgos sistmicos revelan signos vitales e ndices de laboratorio en


deterioro, sndrome de shock y desequilibrio electroltico.
Los hallazgos intestinales y radiolgicos muestran evidencias de perforacin.

Hallazgos radiolgicos en la radiografa de abdomen

Distensin abdominal generalizada

leo paraltico

Neumatosis intestinal quistoide (patognomnico)


La neumatosis intestinal quistoide se la puede visualizar de varias maneras, cmulos de gas
lineales, curvilneos, esponjosos y espumosos.
Este ltimo debe distinguirse de materia fecal o meconio mezclado con aire.
Cualquier lactante con sospecha de enterocolitis necrotizante en el que se encuentren
radiogrficamente colecciones de aire lineales, curvilneas esponjosas o espumosas
debe considerarse que tiene neumatosis intestinal quistoide hasta que se demuestre lo
contrario.
La neumatosis intestinal quistoide suele verse con mayor frecuencia en el colon, pero puede
ocurrir desde estomago hasta recto.

1.

Hartmann G. E., Drugas G. T., Shochat S. J. Post-necrotizing enterocolitis


strictures presenting with sepsis of perforation: risk of clinical observation. J.
Pediatr. Surg. 1988; 23: 562-6.
2. Kosloske A. M., Burstein J., Bartow S. A. Intestinal obstruction due to colonic
stricture following neonatal necrotizing enterocolitis. Ann Surg. 1980 Aug;192 (2):
202-7.
3. Schwartz M. Z., Hayden C. K., Richardson C. J., Tyson K. R., Lobe T. E. A
prospective evaluation of intestinal stenosis following necrotizing enterocolitis. J.
Pediatr. Surg. 1982 Dec; 17 (6): 764-70.


4. Bell M. J., Ternberg J. L., Askin F. B. Intestinal stricture in necroting enterocolitis.
J. Pediatr. Surg. 1976; 11: 319-27.
5. Pokorny W. J., Harr V. L., McGill, C. W., et al; Intestinal stenosis resulting from
necrotizing enterocolitis. Am J. Surg 1981 42: 721-724.

87.-Se trata de paciente de 17 aos de edad que es enviado para valoracin por ortopedia,
el paciente refiere dolor cervical bien delimitado el cual no mejora con AINES, en la
radiografa se aprecia una imagen radiolcida y expansiva de 4 cm de dimetro en el
pedculo de la vrtebra T12. Cul de las siguientes lesiones tumorales ser la ms
probable?

a) Encondroma.
c) Tumor de Ewing.
b) Osteoblastoma.
d) Metstasis de cncer de pulmn.

Osteoteoblastoma Benigno
Llamado tambin osteoide gigante; conformado por clulas de naturaleza osteoblstica,
formador de tejido seo y osteoide, bastante vascular, generalmente de ms de 2 cm de
dimetro; carece de dolor tpico y de hueso reactivo y tiene un cierto potencial de
crecimiento. No es frecuente; ms en el sexo masculino, entre 10 y 25 aos; evidente
preferencia por la columna vertebral, incluyendo el sacro, fmur, tibia, huesos tubulares
cortos de manos y pies.

Clnica
El dolor, generalmente local, es el sntoma cardinal, sin tener la intensidad tpica de las
algias del osteoma osteoide; es producido por la compresin por el tumor, ya sea de mdula


o nervios radiculares y ocasiona a veces paraparesia o paraplejia, otras veces escoliosis,
espasmo muscular y sntomas neurolgicos.
A RX
Zona ltica, radiolucida de 2 a 10 cm. de dimetro rodeada por capa de osteoesclerosis
densa, hueso expandido y engrosado; cuando estn localizados en tejido esponjoso hay
ausencia habitual de osteoesclerosis perifocal.
Anatoma patolgica
Macroscpica: bien limitado, hemorrgico, granuloso y friable; el componente osteoide es
de grado de calcificacin variable; el hueso adyacente no muestra osteoesclerosis, salvo en
los huesos diafisarios, en que hay zona de hiperostosis.
Microscpica: Los osteoblastos con ncleos regulares poco cromatnicos y con abundante
protoplasma producen trabculas entrelazadas o discretos islotes de sustancia osteoide o
tejido seo; hay amplia variacin en la tipologa microscpica, produciendo confusin con los
tumores a clulas gigantes, osteomas osteoides, osteoma y quiste seo aneurismtico; hay
reabsorcin osteoclstica y tambin hueso ms maduro en reconstruccin; nunca se observa
formacin de cartlago; no hay polimorfismo celular, aunque en lesiones jvenes a veces se
observan figuras mitticas que pueden confundir con sarcomas. Al tener una histologa
similar al osteoma osteoide, nadie ha observado que un osteoma osteoide crezca o que un
osteoblastoma haya tenido de inicio un osteoma osteoide. A veces maligniza.
Tratamiento
El de eleccin es curetaje de toda la lesin, seguido de colocacin de injertos si fuese
necesario; si la localizacin lo permite, la escisin en bloque es aconsejable. No es
aconsejable la radioterapia.

Tumores seos en general.

Tumores Benignos de los Huesos

Dr. Luis Julio Huaroto Rosa-Prez


Ciruga: II ciruga ortopdica y traumatologa
Autor: Universidad Nacional Mayor de San Marcos (Lima). Facultad de Medicina. Escuela
Acadmico Profesional de Medicina Humana. Departamento Acadmico de Ciruga
Publicacin: Lima: UNMSM, 2000.


Descripcin: 407 p. : il., tablas, fotos ; 24 cm.
Serie: (Ciruga; 2)
ISBN: 9972-46-102-5
Otros autores: Salaverry Garca, Oswaldo, 1959-, ed.
Tema: Traumatologa; Ortopedia

88.- Masculino de 23 aos, que es referido al servicio de traumatologa por presentar una
tumoracin en hueco poplteo de varios aos de evolucin, lesin que se presenta indolora y
cambia de tamao, su diagnstico clnico es:

a) Quiste de Baker.
b) Hernia muscular de gemelo externo.
c) Sarcoma de partes blandas.
d) Osteocondroma femoral.

El quiste de Baker o quiste poplteo, es una masa de tejido blando localizada en la cara
posterior de la rodilla, la cual se llena con lquido gelatinoso proveniente de la articulacin
de la rodilla. Se ubica con mayor frecuencia del lado interno y posterior de la misma. Se
producen por una debilidad de la membrana sinovial que recubre la articulacin de la
rodilla, generando una hernia de la articulacin.
Desde el punto de vista histolgico se clasifican en:
1. Fibrosos: tienen una pared limitante definida y una superficie interna lisa brillante.
2. Sinoviales: tienen una pared gruesa (2 a 5 mm), es menos brillante.
3. Inflamatorios o Transicionales: Tienen una pared muy gruesa (mas de 5 mm) y su
consistencia es blanda.

En los nios, la molestia inicial es una masa en la regin posterior de la rodilla (hueco
poplteo). Generalmente en nios pequeos, no se observan otros sntomas como rigidez o
dolor local, aunque en adolescentes si se presentan. La masa aumenta de tamao cuando la
rodilla esta en hiperextensin y desaparece cuando la rodilla esta en flexin. La masa es
dura, firme y se puede evidenciar con la transiluminacin (colocar una luz fuerte sobre la
masa, evidencindose la presencia de material lquido).
Es de suma importancia revisar con gran detenimiento la articulacin de la rodilla, con la
finalidad de buscar signos sugestivos de sub luxacin o artrosis. El diagnostico se realiza
cuando se examina al nio, la madre refiere que en ciertas ocasiones, sobre todo despus
de actividad fsica esta de mayor tamao, al levantarse es pequea y aumenta su tamao
durante el da. Se debe realizar un Rx de la rodilla, donde se evidenciara el aumento de los
tejidos blandos y la presencia de una masa de densidad liquida. El ecosonograma ayuda a
delimitar los bordes del mismo. La Tomografa axial computarizada y la Resonancia
Magntica Nuclear sealaran con claridad si el quiste tiene densidad liquida, define la
relacin con el resto de las estructuras y orienta sobre el grosor de la pared del quiste.
El tratamiento inicial es Observacin, ya que el quiste generalmente tiende a desaparecer,
hay que explicarle a los padres la naturaleza de la lesin y que no existe posibilidad de
malignizacin, en trabajos de investigacin se ha determinado que aproximadamente el 80%
de los quistes desaparecen en el transcurso de 2 a 3 aos, en caso de que la masa crezca de
manera importante, que limite la actividad del nio o que comience a presentar dolor, la
indicacin quirrgica no tiene discusin. No se recomienda aspirar el quiste o la infiltracin
con esteroides debido a que con estas tcnicas siempre tiende a reproducirse.

Atlas IRM de Ortopedia y Traumatologa de Rodilla


Autor: Peter Teller Editorial: AMOLCA
N Edicin: 1 Idioma: Castellano

Ao: 2005 N Volmenes: 1

89.- RN 16 meses de edad e inici con tos seca, rinorrea hialina, temperatura axilar de
37.8C. Por la tarde muestra tos intensa y en accesos, dolor subesternal al toser, estridor
inspiratorio; al explorarle se le observa plido, con aleteo nasal, hundimiento intercostal
bilateral a la inspiracin y taquicardia. Faringe hipermica roja; en trax se escuchan
algunos estertores gruesos diseminados en ambos hemitorax y disminucin del murmullo
vesicular.
El agente causal que se asocia ms frecuentemente a este padecimiento es:

a) Virus parainfluenza
b) Staphylococcus aureus.
d) Diplococcus pneumoniae.
e) Streptococcus viridans.

La laringotraqueobronquitis tambin conocida como laringitis subgltica o crup vrico, es


una enfermedad infecciosa aguda de etiologa viral que afecta fundamentalmente a la
laringe y a las estructuras que se localizan por debajo de las cuerdas vocales, en donde los
tejidos blandos subglticos inflamados producen obstruccin de la va area en grado
variable. Este padecimiento afecta con mayor frecuencia al grupo de edad comprendido
entre seis meses y tres aos, con un pico de incidencia a los dos aos de edad; no hay
predominio de sexo, y se presenta ms frecuentemente en otoo e invierno.

ETIOLOGA
Los virus parainfluenza tipos 1, 2 y 3 son los principales agentes causales de la
laringotraqueobronquitis, y con menor frecuencia, los virus sincitial respiratorio,
parainfluenza tipo 2 y adenovirus. En brotes epidmicos, los virus influenza y parainfluenza
tipo 1 son los ms frecuentes. Los cuadros ms graves de la enfermedad se han asociado a
infeccin por parainfluenza tipo 3 e influenza tipo A.

FISIOPATOGENIA
A partir de las superficies epiteliales nasales y farngeas, el virus se extiende por
contigidad hasta afectar la laringe y la trquea, que son las regiones de mayor importancia
clnica; se sabe que 1 mm de edema puede disminuir el dimetro de la subglotis hasta un
65%. Desde estos sitios, la infeccin puede progresar hacia los bronquios, y hasta en un
15% de los casos hacia el intersticio pulmonar.

CUADRO CLNICO
El cuadro inicial es el de una rinofaringitis e incluye irritacin nasal, coriza, fiebre
generalmente menor de 39 C, tos seca y odinofagia, con poca o nula afeccin del estado
general; 24 a 48 horas despus aparece disfona, la tos se hace "crupal" (traqueal, perruna
o en ladrido) y se presentan en forma gradual estridor inspiratorio creciente (estridor
larngeo) y signos de dificultad respiratoria de intensidad variable. En la exploracin fsica
son evidentes la disfona, el estridor larngeo que en ocasiones es audible a distancia, la
disminucin del murmullo vesicular y datos de dificultad respiratoria.1-6.

Forbes describe la progresin del cuadro clnico de la siguiente manera:


Etapa 1: Fiebre, ronquera, tos crupal y estridor inspiratorio al molestar al
paciente.
Etapa 2: Estridor respiratorio continuo, participacin de msculos accesorios de la
respiracin con retraccin de costillas inferiores y de tejidos blandos del cuello.
Etapa 3: Signos de hipoxia e hipercapnia, inquietud, ansiedad, palidez, diaforesis y
taquipnea.
Etapa 4: Cianosis intermitente, cianosis permanente, paro respiratorio.
Por lo general la enfermedad es leve, en pocas ocasiones progresa ms all de la etapa 1,
ms del 95% reciben tratamiento ambulatorio; del 5% que requiere hospitalizacin slo del
1 al 1.5% requieren medidas de apoyo ventilatorio (intubacin endotraqueal o
traqueostoma).
La duracin del padecimiento es muy variable, desde tres a siete das en casos leves, hasta
siete a catorce das en casos graves.

DIAGNSTICO
1. Clnico. Presenta las caractersticas ya mencionadas.
2. Paraclnico. An cuando es de poca utilidad, comprende lo siguiente:
a) Biometra hemtica. Generalmente est normal o presenta leve
linfocitosis.
b) Radiografa lateral de cuello. Con frecuencia muestra sobredistensin de
hipofaringe y estrechamiento de la columna area de la laringe y la trquea
cervical. En estudios dinmicos durante la espiracin se puede observar que
la luz traqueal se reduce.
c) Radiografa posteroanterior de trax. sta
estrechamiento de la luz traqueal en la regin subgltica.

puede

mostrar

3. Etiolgico. Comprende cultivo para virus y serologa para la determinacin de


anticuerpos, slo con fines epidemiolgicos.

DIAGNSTICO DIFERENCIAL
1.

Epiglotitis aguda. No existe perodo prodrmico, se presenta con inicio


sbito y progresin rpida de los signos de dificultad respiratoria, estado
de toxiinfeccin, ausencia de tos crupal y leucocitosis con neutrofilia.

2.

Aspiracin de cuerpo extrao.

3.

Absceso retrofarngeo.

4.

Crup espasmdico.

5.

Crup diftrico.

TRATAMIENTO
El tratamiento se debe individualizar en cada caso de acuerdo con la intensidad de las
manifestaciones de dificultad respiratoria. Es muy importante mantener la calma tanto de
los padres como del personal mdico. La teraputica comprende: 8,9
6.

Medidas generales: Reposo, ambiente tranquilo, control de la temperatura e

hidratacin adecuada.
7.

Permeabilidad de la va area. sta se logra a travs de las siguientes medidas:

a) Ambiente hmedo. En ocasiones con esta medida es suficiente; ms del 90% de los casos
se pueden tratar en casa al proporcionar humedad por medio de una regadera caliente o por
un humidificador fresco en tienda en cama. En el hospital es posible proporcionar
nebulizaciones frescas en una tienda o croupette mediante un aparato humidificador con
30 o 40% de oxgeno y temperatura de 21 a 24 C.
b) Epinefrina racmica. Estimula los receptores alfa adrenrgicos en la mucosa subgltica,
produciendo vasoconstriccin y por lo tanto disminucin del edema de la mucosa. Se


administra al 2.25% en 3 ml de solucin salina al 0.9%, en nebulizaciones y/o sesiones de
presin positiva intermitente (RPPI) durante 15 o 20 minutos; la dosis depende del peso y
es de 0.25 ml en nios con peso menor de 20 kg, 0.5 ml entre 20 y 40 kg y 0.75 ml para

nios con peso mayor de 40 kg; el mximo efecto se obtiene a los 60 minutos despus del
tratamiento; si existe persistencia del estridor y/o datos de dificultad respiratoria a las
dos horas siguientes se debe hospitalizar el paciente. 8-14
c) Traqueostoma o intubacin endotraqueal. Estas medidas son
necesarias en raras ocasiones; slo se utilizan cuando a pesar de
las medidas previas se incrementan los signos de dificultad respiratoria. Si se presenta el
caso, se prefiere la intubacin endotraqueal ya que reduce el tiempo de tratamiento y
hospitalizacin y tiene menor morbilidad. Cuando se requiere la intubacin endotraqueal
debe permitirse que transcurra tiempo suficiente aproximadamente de tres a cinco das
para que remitan la inflamacin y el edema; para realizar la extubacin con mayor xito se
mencionan los siguientes criterios: que el nio est afebril, secreciones escasas, que exista
una fuga de aire alrededor del tubo endotraqueal (presencia de tos o vocalizacin alrededor
del tubo edotraqueal o una fuga audible de aire con una presin positiva de 30 cm de H2O);
se considera que si no existen fugas de aire al cabo de cinco a siete das y se cumplen los
otros criterios, deber extubarse.
3. Esteroides. Su utilidad an es discutible, sin embargo, el uso de una sola dosis
intramuscular o intravenosa de dexametasona a dosis de 0.6 mg/kg ha mostrado
disminucin en la severidad y duracin de los sntomas, con mximo efecto a las seis horas
despus de su administracin y no existe justificacin para repetir la dosis. Si se utilizan
pocos minutos antes de administrar epinefrina racmica se logra potenciar el efecto de
esta ltima. Se ha observado que cuando un paciente requiere intubacin endotraqueal se
logra extubar en menor tiempo si recibe esteroides que si no se le administran
4. Antibiticos. No tienen utilidad y no se deben usar como profilcticos

90.- Masculino de 6 aos acude al servicio de urgencias por salida de scaris por el ano.
A la exploracin fsica presenta dificultad y sibilancia respiratoria, abdomen con
hepatomegalia y distensin abdominal, se confirma el diagnstico por USG.
El tratamiento de eleccin en este paciente es:

a) Secnidazol va oral


b) Metronidazol va oral
c) Salbutamol Inhalado
d) Albendazol va oral

Tratamiento
Se realizar tratamiento farmacolgico por va oral, siendo las drogas de eleccin el
albendazol, el pamoato de pirantel y oxantel y la clsica piperacina
La ascaridiasis intestinal es una patologa ocasionada por un helminto que parasita el tubo
digestivo del ser humano. Es la parasitosis ms frecuente y cosmopolita de todas las
helmintiasis humanas, se calcula ms 1.5 billones de portadores en el mundo de los cuales
51 millones de afectados son nios.

La gran mayora de los casos cursa en forma asintomtico o pueden producirse signos
inespecficos como dolor abdominal difuso o ir acompaado en los casos crnicos por signos
de desnutricin, dficit de crecimiento y retardo del aprendizaje.
Los efectos patolgicos producidos por scaris en el organismo humano, se
presentan en varios sitios de acuerdo a la localizacin de las diversas formas evolutivas.
Las larvas al pasar por el pulmn producen ruptura de los capilares y de la pared alveolar.
Cuando ocurre en forma masiva da origen al sndrome de Leffler que se caracteriza por
lesiones mltiples de los alvolos donde producen granulomas de cuerpo extrao, el cual se
observa a los rayos X como opacidades diseminadas.
Cuando el nmero de parsitos es grande puede ocasionar complicaciones tales como
obstruccin intestinal, vlvulos, y perforacin visceral con peritonitis, complicaciones que
deben ser resueltas en forma quirrgica.

Las mayores complicaciones se presentan por las migraciones de scaris adultos a


diferentes sitios del organismo. La invasin del parsito al rbol biliar, puede causar
ictericia obstructiva, colecistitis acalculosa, colangitis, pancreatitis, abscesos hepticos.


Se han descrito diversas modalidades de tratamiento, La migracin masiva de parsitos
hacia el rbol biliar se asocia con procesos infecciosos, para lo cual deben administrarse
antibiticos aun en forma profilctica. La administracin de albendazol se considera el
tratamiento antihelmntico de eleccin por su mayor excrecin biliar. En un bajo
porcentaje (21.2%) se logra expulsar los parsitos de la vescula biliar. Si no se logra
expulsarlo, si persiste la sintomatologa o si el paciente empeora, es necesario someterlo a
una laparotoma exploradora para realizar colecistectoma.

Los pacientes pueden caer en una septicemia fatal, si no llevamos a cabo la teraputica en
forma adecuada, lo que hace que la evolucin y el pronstico empeoren.

La parasitosis es uno de los grandes problemas de la salud pblica. La OMS la considera una
de las principales causas de morbilidad; estrechamente ligada a la pobreza y relacionada
con inadecuada higiene personal y de los alimentos crudos, falta de servicios sanitarios,
falta de provisin de agua potable y contaminacin fecal del ambiente. Infecta a personas
de todas las edades, pero la sufren principalmente los nios, a lo que les causa trastornos
del crecimiento y desarrollo.

Desde hace aos la OMS propone como solucin, aunque sea parcial, el uso de tratamientos
antihelmnticos masivos y reiterados, sin diagnstico parasitolgico previo individual, en
aquellas comunidades que tengan una elevada prevalencia de geohelmintos (>50%).

Bibliografa:
Cook GC, Zumla AI (edit). Mansons Tropical Diseases. 21th ed. London: Elsevier Science;
2002.
Garca LS. Diagnostic Medical Parasitology. 4th. Ed. Washington: American Society for
Microbiology; 2002.
Gill G. Lectura Notes on Tropical Medicine. 5th ed. Blackwell Scientific Pub; 2004.
Mensa JM, Gatell MT, Jimnez de Anta G, Prats A, Dominguez-Gil A. Gua terapetica
antimicrobiana. 14 ed. Barcelona: MASSON; 2004.


Murray PR, Baron EJ (Edit.)Manual of clinical microbiology. 8th edit. Washington: ASM
Press; 2003.
Villa Luis F. Gua de Terapia Farmacolgica. Medimecum. Barcelona: ADIS Internacional;
2005.

91.- Femenino de 19 aos con 7episodios al mes de cefalea hemicraneal, de un da de


duracin, pulstil, acompaada de vmitos, fotofobia y sonofobia y con examen fsico
normal. El tratamiento preventivo de eleccin, entre los siguientes, es:

a) Carbonato de litio.
b) Ergotamina.
c) Propranolol.
d) Sumatriptn.

Profilaxis de Migraa
Debe ser considerada en aquellos pacientes que tienen 2-3 ataques en el mes o en aquellos
que tienen 1 ataque pero prolongado. La profilaxis puede ser necesaria 6-9 meses e incluso,
en forma definitiva.
Los principales agentes utilizados son los Beta bloqueadores adrenrgicos Propanolol,
Atenolol... (Excepto asmticos, insuficiencia cardaca, diabetes). En un estudio controlado
se demostr una reduccin de migraa en relacin con placebo de 76%.
Bloqueadores de los canales del Calcio Nifedipino Diltiazem... Vasodilatadores, Fluranizina.
REFERENCIAS BIBLIOGRAFICAS
1.
2.
3.
4.

Lewis PJ Neurology 1997 48 (6) 1542-50.


Mathew NT Neurol Clin. 1997; 15:61-83.
Goadsby PJ Neurol Clin 1997; 15 (1[):27-42.
MLTfeld-Hansen P Neurol Clin 1997; 15: 153-165.


5. Diamond S., Diamond ML Postgrad Med. 1997; 10: 169-172.

92.- Femenino de 22 aos de edad, acude al servicio de urgencias por referir malestar
general, fosfenos, nusea y vmito. Antecedentes: cursa con embarazo de 36.5 SDG, G2,
A1, C0. Exploracin fsica: TA 185/110 mmHg, FC 120 lpm, FR 35 x, T37.9, alerta,
inquieta, aprecia ictericia, cardiopulmonar sin compromiso, abdomen globoso a expensas de
tero grvido, se detecta PUVI, longitudinal, ceflico, dorso a la derecha, FCF 142 x, al
tacto genital crvix, central, formado, cerrado, extraccin de guantes sin evidencia de
prdidas genitales, extremidades inferiores edema +++.
La medida teraputica inicial en esta paciente es:

a)
b)
c)
d)

Diurticos.
Sedantes.
Antihipertensivos.
Analgsicos.

Referencia:
La preeclampsia se define por aumento de la presin arterial y la presencia de proteinuria
durante el embarazo.
5) Leve: las pacientes suelen tener unas cuantas manifestaciones, y su presin arterial
diastlica es menor de 110 mmHg. En ocasiones hay edema. La cifra de plaquetas es
mayor de 100 000/l.
6) Grave: los sntomas son ms notorios y persistentes. La presin arterial casi
siempre es con niveles mayores de 160/110 mmHg. Puede haber trombocitopenia
que avance hasta coagulacin intravascular diseminada.
Se requiere hospitalizacin para las mujeres con preeclampsia; debe obtenerse una
biometra hemtica completa con cifra de plaquetas y determinacin de electrolitos, que
adems incluya enzimas hepticas. Se obtiene una muestra de orina de 24 horas para
determinar la depuracin de creatinina y protenas totales al ingreso hospitalario.


Se debe controlar la hipertensin arterial, para evitar sufrimiento fetal, as como
empeoramiento de la paciente.

Bibliografa:

1.McPhee S, Papadakis M, et. al. Diagnstico Clnico y Tratamiento 2010. Lange, McGraw
Hill,
49 edicin, Mxico, 2010.
2.Sibai BM, Diagnosis, prevention, and management of eclampsia. Obstet Gynecol. 2005.
Feb; 105;: 402 410.

93.- En un nio de 4 aos se observan inicialmente fiebre baja, cefalea y mucosidad nasal.
Se sospecha una sinusitis, pero la transiluminacin de los senos y un cultivo de garganta no
sirven para establecer este diagnstico. Una radiografa de senos muestra infamacin
bilateral de la membrana sinusal maxilar. Cul de las siguientes afirmaciones sobre el
diagnstico de sinusitis del nio es la ms precisa?

a) La transiluminacin de los senos rara vez ofrece informacin valiosa para el


diagnstico de sinusitis en nios pequeos.
b) Los cultivos obtenidos con torunda de la garganta suelen ser fiables para el
diagnstico de sinusitis.
c) La evidencia de inflamacin de la membrana sinusal maxilar bilateral en la
radiografa confirma el diagnstico de sinusitis bacteriana aguda.
d) Rara vez se necesita puncin para confirmar el diagnstico de sinusitis.


La puncin de los senos no es necesaria para confirmar el diagnstico de sinusitis. La
transiluminacin de los senos proporciona buena informacin para el diagnstico pero no es
fiable, as como el cultivo de faringe pues puede haber contaminacin de la muestra. Como
la mucosa de los senos paranasales se contina con la de la nariz, incluso las infecciones
virales pueden provocar inflamacin de la membrana sinusal lo que aparece como densidades
difusas en la radiografia. Un nivel unilateral o una opacidad completa de un seno, sugiere
una sinusitis bacteriana aguda.

94.- Se trata de femenino de 57 aos, cursa con cuadro febril de cuatro semanas de
evolucin, se diagnostica toxoplasmosis. En las tres semanas previas haba recibido ampicilina oral a la dosis de 2g/da, con lo que haba descendido algo, sin desaparecer, la
fiebre. Unos das antes del ingreso vuelve a elevarse la fiebre, aparece eritema, oliguria de
500 a 700 mL/da, creatinina plasmtica de 4,3 mg/dL y hematuria macroscpica con
proteinuria de 2g/24 h. En el sedimento el 80% de los hemates estn deformados y hay
algunos eosinfilos. Los niveles de C3 y C4 en plasma son normales. El cuadro revirti al
final, dejando una filtracin glomerular del 75%. El diagnstico ms probable es:

a)
b)
c)
d)

Necrosis tubular aguda.


Nefritis intersticial aguda por hipersensibilidad
Glomerulonefritis mesangiocapilar tipo II (depsitos densos).
Glomerulonefritis membranosa.

NEFRITIS INTERSTICIAL AGUDA


La prevalencia de nefritis intersticial aguda (NIA) en la poblacin sana sometida a biopsia
renal para estudio de hematuria o proteinuria es del 1%1 aunque se han descrito
prevalencias que oscilan entre 1-15% en estudios de biopsias renales de pacientes que
presentaban enfermedad renal aparente2.
Aunque la NIA es con frecuencia inducida por frmacos, se ha descrito asociada a otras
enfermedades como sarcoidosis, infeccin por Legionella, Leptospira estreptococos e
infecciones virales3.
Los cambios histolgicos ms relevantes son edema e infiltrado intersticial por linfocitos y
monocitos; se observan eosinfilos, clulas plasmticas y neutrfilos.


La formacin de granulomas, caracterstica de la sarcoidosis, aparece en cualquier forma
de NIA4.

FORMA DE PRESENTACIN
Este trastorno se manifiesta con las siguientes caractersticas:
Aumento brusco de la creatinina srica.
Fiebre en la mayora de casos asociada a exantema cutneo.
Sedimento urinario con leucocituria, hematuria y cilindros leucocitarios.
Eosinofilia y eosinofilinuria en > 75% de casos1 a excepcin de la NIA inducida por
antinflamatorios no esteroides (la fiebre, exantema y eosinofilia suelen estar ausentes).
Proteinuria ausente o discreta (< 1 g/da). Sin embargo, en algunos pacientes con NIA
atribuible a antinflamatorios no esteroides, ampicilina o rifampicina se asocia proteinuria
de rango nefrtico en el contexto de enfermedad por cambios mnimos.
Signos de lesin tubulointersticial, como sndrome de Fanconi y acidosis tubular renal.
ETIOLOGA
Aunque la lista de frmacos que se asocia a NIA es muy extensa, slo merecen
considerarse los ms importantes:
Meticilina (de poco uso en la actualidad, aunque anteriormente prevalencia
17%).
Antinflamatorios no esteroides (AINE).
Penicilinas y cefalosporinas.
Rifampicina.
Sulfonamidas (furosemida, bumetanida, diurticos tiacdicos, sulfametoxazol-


trimetoprim).
Cimetidina.
Alopurinol.
Ciprofloxacino y otras quinolonas.
Sulfasalazina.

La instauracin de NIA tras la administracin del frmaco responsable oscila entre 3-5
das para una segunda exposicin al frmaco y varias semanas tras una primera exposicin4.
Sin embargo, el perodo de latencia puede ser tan corto un da como en el caso de la
rifampicina, o tan largo 18 meses como en los AINE5.
DIAGNSTICO
Antecedente, ingesta frmacos
Alteraciones
Sedimento urinario
Eosinofilinuria
Alteracin mnima
Sedimento urinario
Escasas clulas/cilndricas
Intervalo tiempo entre frmaco
y alteracin sedimento
Sedimento casi normal
Fraccin excrecin sodio > 1%
Un anlisis de orina relativamente normal no descarta la NIA. El intervalo de tiempo entre
la ingesta de un frmaco y los hallazgos en el sedimento urinario permiten distinguir la NIA
de otras causas de insuficiencia renal, como: Necrosis tubular aguda: c. granulosos/c.
epiteliales; glomerulonefritis: hematuria/


Aunque el cuadro clnico sea muy indicativo de NIA, el diagnstico slo puede confirmarse
por biopsia renal. Sin embargo, en ausencia de insuficiencia renal importante se adoptar
una conducta expectante, tras la retirada del frmaco.
No se precisarn ms exploraciones ni tratamiento si la funcin renal mejora en pocos das.
El diagnstico confirmativo de NIA requiere la prctica de biopsia renal en caso de:
incertidumbre diagnstica, insuficiencia renal importante o ausencia de recuperacin
espontnea tras retirada del frmaco responsable.

En pacientes poco candidatos a biopsia renal, est indicado iniciar un tratamiento con
esteroides (1 mg/kg/da).
En los pacientes con NIA, la funcin renal empieza a mejorar dentro de los siete das; se
observa un rpido retorno a los niveles de creatinina basal6. El papel de la gammagrafa con
galio en el diagnstico de NIA no est bien definido dada la elevada frecuencia de falsos
negativos.

BIBLIOGRAFIA:
1. Elseviers MM, Bosteels V, Cambier P et al. Diagnostic criteria of analgesic
nephropaty in patients with end-stage renal failure: results of the Belgian Study.
Nephrol Dial Transplant 1992; 7:479-486.
2. Morlans M, Laporte JR, Vidal X, Cabeza D, Stolley PD. End-stage renal disease and
non-narcotic analgesics: a case-control study. Br J Clin Pharmac 1990; 30:717723.
3. Dubach UC, Rosner B, Pfister E. Epidemiologic study of abuse of analgesics containing
phenacetin: renal morbidity and mortality (1968-1979). N Engl J Med 1983;
308:357-362.

95.- Se trata de paciente femenino de 6 aos de edad la cual presenta ceguera nocturna
(hemeralopia) en la exploracin oftalmolgica se observa constriccin del campo visual con
escotoma anular, prdida de la agudeza y electrorretinograma anmalo asi mismo arterias
estrechadas Cul de las siguientes patologas es la ms probable?

a) Retinoblastoma.
b) Catarata congnita o infantil.
c) Retinosis pigmentaria (retinitis pigmentaria).
d) Persistencia de vtreo primario

Retinosis pigmentaria. Bilateral, de curso lento y progresivo, comienza en la edad escolar,


pudiendo causar ceguera hacia los 40 aos. Es una alteracin de los bastones. Puede
presentarse aislada o asociada a otras malformaciones, como la polidactilia, el sndrome de
Laurence-Moon-Bield.
Se hereda bajo diferentes patrones. Clnicamente presentan mala visin nocturna
(hemeralopa) y escotoma anular en el campo visual .Oftalmoscpicamente se aprecian
arterias estrechadas, atrofia de papila y acumulaciones de pigmento que asemejan
osteocitos en la retina perifrica. Evolucionan hacia una reduccin del campo visual y
finalmente ceguera. Adems presentan complicaciones adicionales como aparicin temprana
de cataratas y glaucoma.

Retinosis pigmentaria

Retinosis pigmentaria, preguntas y respuestas, Universidad Miguel Hernndez de Elche,


2007
Science Daily (ed.): Retina Transplants Show Promise In Patients With Retinal
Degeneration (11-7-2008).

96.- Los tumores malignos que surgen en tejidos mesenquimales se les denomina:
a) Hamartomas
b) Carcinomas
c) Sarcomas
d) Teratomas

Neoplasias malignas mesenquimatosas. Los tumores malignos que nacen en los tejidos
mesenquimales suelen denominarse sarcomas, del griego sar (carnoso), porque en general
poseen muy poco estroma conjuntivo y en consecuencia su estroma es blando. El esquema
utilizado para su denominacin es el mismo que en las neoplasias benignas con la utilizacin
del sufijo sarcoma ejemplo, fibrosarcoma, condrosarcoma y leiomiosarcoma.
Tejido de origen

Benigno

Maligno

Tejido fibroso

fibroma

fibrosarcoma

Tejido seo

osteoma

osteosarcoma


Tejido cartilaginoso

condroma

condrosarcoma

Msculo liso

leiomioma

leiomiosarcoma

rabdomioma

rabdomiosarcoma

lipoma

liposarcoma

Vasos sanguneos

hemangioma

hemangiosarcoma

Linfticos

linfangioma

linfangiosarcoma

Msculo estriado
Tejido adiposo

Hay excepciones como las neoplasias linfoides malignas que se denominan Linfomas. Las
originadas en los melanocitos: Melanomas.

97.- Femenino de 66 aos que acude al servicio de neurologa para valoracin, al realizar
exploracin fsica, usted determina que ste, comprende pero no puede articular palabra,
Que tipo de afasia tiene?

a) Wernike
b) Broca
c) Global
d) Anmica

Afasia

De broca, motriz o de expresin (no fluente)


De Wernicke (no comprende)
De conduccin (no denomina no repite)
Anmica (no denomina)
Transcortical motora (no fluente)
Transcortical sensitiva (no denomina no comprende)
Global (no fluente-no denomina-no repite-no comprende)
Talmicas y putaminales (lenguaje espontaneo incomprensible)

1.Aronson A y cols. Examen clnico neurolgico, 3 Edicin. La Prensa Mdica Mexicana,


Mxico, 1995.

2. Uribe CS, Arana A, Pombo PL. Neurologa, 5 Edicin. Corporacin para


investigaciones biolgicas. Colombia, 1996.

3.-Adams R, Vctor M. Principles of Neurology. Mc Graw Hill. 7th ed. USA 2001.

4.-Bradley W.G. Neurology in clinical practice. Butterworth Heinemann. 4th ed.


Philadelphia USA, 2004.

5.-Silberstein SD, Lipton RB, Goadsby PJ. Headache in clinical practice. Isis Medical
Media. Oxford University Press, UK, 1998.

6.-Barinagarrementera F. Cant C. Enfermedad vascular cerebral. McGrawHill


Interamericana. Mxico D.F. 1997.

98.- Femenino de 19 aos que acude al servicio por datos de amenorrea secundaria se
indica tratamiento basado en progestgenos, posterior a ello la paciente inicia con
menstruacin, lo ms probable es que tenga?

a)
b)
c)
d)

Estrgenos bajos
Estrgenos normales
Progesterona elevada
Gonadotropinas altas


La presencia de estrgenos se puede establecer de dos maneras: con la prueba de desafo
con acetato de medroxiprogesterona (AMP) y el ndice de maduracin vaginal. Si hay
estrgenos, luego de la administracin de 10 mg de AMP diarios por 10 das debera
presentarse un sangrado por vagina. Y de esta forma corroborar los niveles de estrgenos
serian normales.
La ausencia de esta metrorragia sugiere una obstruccin o un
hipogonadismo. La otra alternativa es realizar el ndice de madurez vaginal que es un
procedimiento sencillo que puede obtenerse en el consultorio. Usando una esptula como las
de Papanicolaou el gineclogo extrae una muestra de la vagina y la extiende en un
portaobjeto. Este portaobjeto se procesa y se estudia en el microscopio. Un extendido
maduro tendr clulas epiteliales superficiales grandes y numerosas con un ncleo rodeado
por una gran cantidad de citoplasma. Por el contrario, una muestra no estrognica tendr un
mayor nmero de clulas parabasales y basales con ncleo grande rodeado por escaso
citoplasma.

Speroff L, Glass RH, Kase NG, eds. Clinical Gynecologic Endocrinology and Infertility, 5th
ed. Baltimore: Williams & Wilkins; 1994:334-335.

99.- Monte verde es una comunidad de 100,000 personas. Durante 2001 hubo 1,000
defunciones por todas las causas. Durante el mismo ao se registraron un total de 300
casos de Infartos Agudos al Miocardio y 60 defunciones por esta misma causa. La tasa de
mortalidad bruta en Monte verde es:
a)
b)
c)
d)

100 por 1,000


10 por 1,000
300 por 1,000
150 por 1,000

La tasa de mortalidad bruta mide la proporcin de la poblacin que muere cada ao o el


nmero de muertes en la comunidad por 1, 000 habitantes. Se calcula dividiendo todas las
muertes de un ao entre la poblacin total por 1, 000. ( TM= 1,000 / 100, 000 por 1, 000)

Morton R. F. Bioestadstica y Epidemiologa, Interamericana, 3. Ed. 1993; pg: 19.

100.- Cul es la causa ms frecuente en el adulto de insuficiencia testicular adquirida?

a)
b)
c)
d)

Orquitis viral
Orquiectoma
Cncer testicular
Disgnesia gonadal

La orquitis vral puede deberse al virus de la parotiditis epidmica, virus ECHO, virus
linfoctico de la coriomeningitis y arbovirus del grupo B. La orquitis ocurre hasta en una
cuarta parte de los varones adultos que sufren parotiditis. Es unilateral en casi dos tercios
y bilateral en los restantes. El anlisis del semen solamente se normaliza en 33% de los
varones que sufren orquitis bilateral.
Bashin S, Jameson JL. Trastornos de los testculos y del sistema reproductor masculino. En
Jameson JL (ed): Harrison. Endocrinologa. 1a ed. Madrid. MacGraw-Hill Espaa, 2006: 184.

Potrebbero piacerti anche